[HN Gopher] An intutive counterexample to the axiom of choice
       ___________________________________________________________________
        
       An intutive counterexample to the axiom of choice
        
       Author : lisper
       Score  : 96 points
       Date   : 2023-01-23 12:17 UTC (10 hours ago)
        
 (HTM) web link (blog.rongarret.info)
 (TXT) w3m dump (blog.rongarret.info)
        
       | [deleted]
        
       | mannykannot wrote:
       | I suspect this posting might have been prompted by the recent one
       | of Colin Wright's "The Point of the Banach-Tarski Theorem."[1]
       | Regardless, Colin has also written about the axiom of choice, in
       | "In Defense Of The Axiom Of Choice"[2] and "A Point Against The
       | Axiom Of Choice."[3]
       | 
       | [1]
       | https://www.solipsys.co.uk/new/ThePointOfTheBanachTarskiTheo...
       | https://news.ycombinator.com/item?id=34482226
       | 
       | [2]
       | https://www.solipsys.co.uk/new/InDefenseOfTheAxiomOfChoice.h...
       | 
       | [3]
       | https://www.solipsys.co.uk/new/APointAgainstTheAxiomOfChoice...
        
       | [deleted]
        
       | [deleted]
        
       | rtpg wrote:
       | since everyone here is bike shedding about the axiom of choice...
       | I have this axiom of choice book (by Thomas Jech). It includes
       | the following:
       | 
       | > if F is a finite family of nonempty sets, then there is a
       | choice function (to show this one uses induction on the size of
       | F; naturally a choice function exists for a family which consists
       | of a single nonempty set).
       | 
       | could someone lay out how one would "correctly" describe a choice
       | function for a family with a single nonempty set? I feel like I
       | understand theoretically this idea that choosing isn't about
       | providing the value, but some... logical instantiation? What is
       | the piece of the puzzle I'm missing here to understand why the
       | choice function exists "naturally"?
        
         | hgsgm wrote:
         | Sets are defined by their members. If you have a nonempty set,
         | you must have defined a member when you proved it was nonempty.
         | Take any convenient one.
         | 
         | This works for any finite or countable family, because you can
         | repeat the process for each set, any set you can think of will
         | be processed eventually.
         | 
         | But for an uncountable set, _almost every_ set will _never_ be
         | processed, and _almost every_ set is impossible to describe in
         | finite time (per set!), unless you just assume that you can do
         | it on one magical oracle bulk selection.
        
           | rtpg wrote:
           | Is the correct way to see this is that logic has to be
           | specified in a finite number of terms? For countable families
           | there's some sort of meta logic like "for the Nth element I
           | induct to that"?
           | 
           | I believe to understand that idea (since it comes up in real
           | analysis to some extent, basically acknowledging there isn't
           | really infinite sums except as limits of finite sums), but is
           | there a specific sort of text that lays out this idea
           | specifically?
        
         | quchen wrote:
         | Good question, I'm confused about this as well! Take for
         | example the single-element set containing the set of R-R
         | functions, so { {f | f: R-R} }. The author seems to suggest
         | there's a natural choice for those functions. To me this sounds
         | as hard as choosing from a larger set-of-sets.
        
           | housecarpenter wrote:
           | I don't think Jech is using "naturally" as in "natural
           | choice"; he's just using it as a synonym for "obviously". If
           | X is non-empty set then there exists an x in X, because
           | that's what it means for X to be non-empty. The choice is
           | arbitrary: if X is the set of all functions from R to R then
           | you could take the exponential function, the x^2 function,
           | etc., whatever you like; the point is that it is certainly
           | possible to make _some_ choice.
           | 
           | Doubting the axiom of choice means thinking that when you
           | have infinitely many sets to choose from, it is possible that
           | not only is there is no _natural_ choice function, but there
           | is _no choice function at all_.
        
             | rtpg wrote:
             | But why do we need the axiom of choice at all? By this
             | logic I have any family of non-empty sets then I'm good?
             | But surely there's a distinction here
        
       | Aardwolf wrote:
       | But even if you have the set of all reals not describable with a
       | finite amount of symbols, I can still imagine what happens e.g.
       | if you multiply all its values by 2, or what the cartesian
       | product of this set with another is like. At least, I can reason
       | sbout that result in a way that's not worse than reasoning about
       | the set itself. Using axiom of choice on it does not make things
       | harder than the set itself already is
        
         | hgsgm wrote:
         | AoC lets you ignore how bad the set it.
         | 
         | Banach-Tarski says: Take a measurable set. Split it up into 2
         | unmeasurable sets. Recombine them into a set with twice the
         | measure of the original.
         | 
         | OK, you can do that, but you've lost the ability to make any
         | reasonable theorems about measure of measurable sets, unless
         | you add qualifications of the form "this theorem does not apply
         | if you introduce a set of uncountable sets" which. What's the
         | benefit? To get a bunch of "proofs" for things that are only
         | true because of the axiom, and don't model anything else we
         | care about?
        
       | voldacar wrote:
       | The axiom of choice is not really about assigning names to
       | objects using finite strings of symbols. In this example, you
       | take R, remove countably infinitely many elements, and end up
       | with a set with uncountably many elements. From a set theoretic
       | perspective, it is not any more difficult or unintuitive to
       | select an element from this set using AoC than it would be for R.
       | When we use AoC, we are not really assigning a name to anything
       | or doing any computation. We merely claim that a function exists
       | which returns some element from the set. AoC doesn't compute or
       | describe this function.
        
         | horsawlarway wrote:
         | > We merely claim that a function exists which returns some
         | element from the set.
         | 
         | Isn't this where the challenge lies, though?
         | 
         | What if you have a set for which there is no function which
         | returns some element from the set, but the set is non-empty?
        
           | hgsgm wrote:
           | It's only a challenge if you want to actually do it.
           | Mathematicians figured out that it's impossible to actually
           | do, so they decided to pretend it's possible, because nothing
           | else breaks if they just pretend it's possible.
           | Mathematicians like pretending.
        
           | drdeca wrote:
           | For any set S which is non-empty, there is an element x of
           | that set. For any set S and any x, the set S * {x} is a
           | function which sends every element of S to x.
           | 
           | Therefore, for every non-empty set S, there is a function
           | with domain S, and range a single element of S.
           | 
           | Did you mean a set S of sets such that there is no f : (A in
           | S) -> A ?
        
       | raldi wrote:
       | Even more intuitive: What's the first positive real number?
       | 
       | If you can't tell me that, or even be sure that it even exists,
       | how can you be sure you can identify the first item in any set?
        
       | Chinjut wrote:
       | As others have noted, this has little to do with the Axiom of
       | Choice per se. The Axiom of Choice is not needed to prove that
       | one can make a choice from a single inhabited set, where this
       | boils down to the tautology "If a set has at least one element,
       | then it has at least one element". The Axiom of Choice is only
       | needed for making infinitely many choices at once.
       | 
       | (There IS a sense in which what is going on here is the
       | distinction between what category theorists call "internal" and
       | "external" Choice, but I'll not get into that for now since I
       | think the following is more important to point out first.)
       | 
       | The result/paradox here has more to do with Tarski's
       | indefinability theorem, under which no suitable formal language
       | is able to define the concept of definability in that same
       | language.
       | 
       | See also https://mathoverflow.net/a/44129/3902 for more
       | discussion on what goes wrong in attempting to formally prove
       | that "undefinable" numbers exist by this common but naive
       | argument from uncountability.
       | 
       | The gist of the above MathOverflow link and of Tarski's
       | indefinability theorem is that we must keep in mind that Cantor's
       | famed diagonal argument for uncountability is a constructive
       | argument. Given any function from N to 2^N, it defines an
       | explicit element of 2^N not in the range of that function. So
       | there would be something paradoxical in concluding that this
       | explicitly defined value is not definable.
        
       | malaya_zemlya wrote:
       | A slightly more elaborate setup which showcases the weirdness of
       | the Axiom of Choice.
       | 
       | Consider a group of countably infinite wizards w_1, w_2,... A
       | king has put a number 0 or 1 on their backs and ordered the
       | wizards to stand in a single file, such that wizard w_n can only
       | see the numbers on the backs of all wizards with higher numbers
       | (w_n+1, w_n+2...). As is the tradition, the king gave them a
       | challenge: if you can guess the number on your back, your life is
       | spared, otherwise off with your head. The wizards can discuss
       | their actions beforehand, but no communications is allowed after
       | the numbers have been assigned.
       | 
       | The wizards cannot see their own numbers, they cannot exchange
       | information, yet, if we assume the Axiom of Choice, they still
       | can work out a strategy, such that all but a finite number of
       | wizards survive.
       | 
       | The trick is to break the space of all 0-1 sequences into
       | equivalence classes, whre X is equivalent to Y if they disagree
       | only in finite number places. (in other words, they fully agree
       | starting from some index k). That's an equivalence relation, as
       | you can easily verify.
       | 
       | Then the wizards use the Axiom of Choice to remember one
       | representative sequence from each class (there are infinitely
       | many such sequences, but our wizards are infinitely wise). During
       | the test, each wizard would look at what numbers they see in
       | front of them, pick the representative sequence that is
       | equivalent to what they are seeing, and guess the number that
       | corresponds to their place in the sequence.
       | 
       | All wizards will agree on what representative sequence will need
       | to be picked (because the king's sequence belongs to a single
       | equivalence class), and that sequence will agree with the king's
       | sequence everywhere except for a finite number of places. Boom!
        
         | voldacar wrote:
         | >Then the wizards use the Axiom of Choice to remember one
         | representative sequence from each class
         | 
         | I don't think AoC is enough here becuase the wizards need to
         | evaluate the choice function to get the representative from
         | each equivalence class. AoC merely states that there is some
         | function that maps each equivalence class to some element
         | within that equivalence class. It doesn't tell you how to
         | actually evaluate it, which is what the wizards need to do.
        
         | CaptainNegative wrote:
         | What's the role of restricting the number domain to binary
         | here? Can't they each have a real-valued number on their back
         | (or worse, something from a larger set) with the same argument
         | still going through?
        
           | IngoBlechschmid wrote:
           | Indeed, it works as long as the class of labels forms a set
           | instead of a proper class.
        
       | hackandthink wrote:
       | I would pick Chaitin's constant.
       | 
       | It is not only transcendantal like Pi, it is even uncomputable,
       | so it would be part of his counterexample set.
       | 
       | (Pi is computable, so you can describe it with an algorithm
       | (finite number of characters))
        
         | moron4hire wrote:
         | But can't Chaitin's constant be described using a finite set of
         | symbols, those symbols being in the set [achinost'] (at least,
         | in English)?
        
           | hackandthink wrote:
           | OK, original poster doesn't speak about computable numbers:
           | "consider the set of numbers that cannot be described using
           | any finite collection of symbols"
           | 
           | But I have a hard time accepting numbers, which can only be
           | described in natural language.
        
           | tromp wrote:
           | Sure; a simple definition is at https://tromp.github.io/cl/Bi
           | nary_lambda_calculus.html#Halti...
        
       | _hark wrote:
       | These kinds of non-constructive results are almost always present
       | when you invoke choice (when it's actually relevant), and this is
       | a well-known example of the non-intuitiveness of choice. If you
       | don't like this, you're probably a constructivist and may also
       | disagree with the law of the excluded middle. A classic
       | quote/joke on the topic:
       | 
       | "The Axiom of Choice is obviously true, the well-ordering
       | principle obviously false, and who can tell about Zorn's lemma?"
        
       | NotYourLawyer wrote:
       | > the set of numbers that cannot be described using any finite
       | collection of symbols
       | 
       | Not clear that this is a well-defined set.
        
         | hgsgm wrote:
         | It's probably not well-defined.
         | 
         | https://en.m.wikipedia.org/wiki/Richard's_paradox
        
         | [deleted]
        
       | sylware wrote:
       | If I recall properly, at the time I was "building maths from the
       | ground up (aka axioms)", I was very uncomfortable with THE
       | axioms: this mixing between discret/finite concepts/objects and
       | infinite concepts/objects was too easy for my taste.
        
       | phkahler wrote:
       | I've always called BS on the math community for this. Let's talk
       | about real numbers between. 0 and 1 (because we can map all of
       | them to this range anyway). I can (count) enumerate all of them.
       | Simply flip the digits over the decimal point, so 0.14159 become
       | integer 95141. Done. Now you ask where pi sits? It's infinitely
       | far down the list of integers. It's position is as precise as
       | your definition.
       | 
       | We can extend this to (I forget the terminology) enumerate the
       | numbers that can be named, by converting the names to integers.
       | Now pi has several location in this mapping, as do a lot of real
       | numbers.
       | 
       | Infinite sets are interesting, but I've never been convinced that
       | there are more reals than integers. It seems plausible but I
       | don't think pi not mapping to a _specific_ integer in the above
       | scheme invalidates the mapping.
        
         | 101008 wrote:
         | If you sort this set after you apply your transformation,
         | what's the number that comes right after pi?
        
           | dsr_ wrote:
           | You can begin looking for it knowing just two facts: the last
           | digit of pi, and the magnitude of that digit.
           | 
           | Easy!
           | 
           | [sarcasm]
        
             | phkahler wrote:
             | This is all related to the ability to name a specific
             | number. If there are reals that can't be named in a finite
             | number of symbols, why can't there be integers like that as
             | well?
        
               | davrosthedalek wrote:
               | Because a property of the numbers we call integers is
               | that we can name them. You can certainly rename some of
               | the numbers which are indescribable and call that set
               | your integers, but these are different integers than what
               | everyone else talks about.
        
         | chriswarbo wrote:
         | > Simply flip the digits over the decimal point, so 0.14159
         | become integer 95141
         | 
         | This sounds similar to an interesting (and often counter-
         | intuitive) way of writing down numbers called 'p-adic numbers'
         | (although that terminology usually requires 'p' to be prime; in
         | your case you'd want the '10-adic' numbers, since you're using
         | base-10)
         | 
         | See e.g. https://en.wikipedia.org/wiki/P-adic_number (here's a
         | video specifically about 10-adic numbers
         | https://www.youtube.com/watch?v=XXRwlo_MHnI )
        
         | alecbz wrote:
         | Don't want to pile on too much, but my attempt at an
         | explanation:
         | 
         | > Simply flip the digits over the decimal point, so 0.14159
         | become integer 95141
         | 
         | They key here is that both 0.14159 and 95141 contain a _finite_
         | number of symbols/digits. There are infinitely many numbers
         | like this, but each of those numbers are themselves finite.
         | 
         | The decimal expansion of pi, OTOH, is _itself_ infinite. The
         | decimal expansion of any irrational number is infinite. There
         | are infinitely many of these things, each of which are
         | themselves infinite.
         | 
         | This is the key thing that causes uncountability. Each of the
         | numbers has infinitely many "parameters" that can be tweaked.
         | If you try to produce an infinite (countable) list that
         | contains all of these infinite numbers, I can always tweak a
         | different parameter for each number in your list and be
         | guaranteed to have produced a number that your list missed.
        
         | iroddis wrote:
         | "Flip around the decimal"... you haven't accounted for
         | irrational numbers. The reals are not countably infinite.
         | 
         | Even if they were, pi is not between zero and one, so would not
         | appear in your list.
         | 
         | The rest of your argument is difficult to parse. If you're
         | interested in this kind of stuff though, I'd really recommend
         | picking some good books on set theory and maybe number theory.
         | Set theory in particular will cover a lot of what you're
         | talking about.
        
         | [deleted]
        
         | SantalBlush wrote:
         | You can't flip irrationals over the decimal the way you
         | described, because their decimals have infinitely many non-
         | repeating values. You would end up with an infinite sequence of
         | numbers, not a real number.
        
           | phkahler wrote:
           | You mean there aren't infinity large integers? Hmm, that
           | seems weird.
        
             | xvkektjdicou wrote:
             | All integers are by definition finite. Positive integers
             | are defined by S(S(S(...S(0)...))) a finite number of
             | times, where S is the successor function. (And negatives in
             | the obvious way.)
        
         | SamBam wrote:
         | Your method is not a 1:1 mapping of reals to integers, because
         | you can't even define the position of a single irrational
         | number with your method. You don't know the final digit, so you
         | don't know the leading digit in your system. You have
         | absolutely no idea the ordering of literally any pair of
         | irrational numbers in your system.
         | 
         | > It's position is as precise as your definition.
         | 
         | This wand-waving is actually transforming irrational numbers
         | into rational numbers, and so means you are not actually
         | working with irrational numbers. You can't just say "this works
         | with all irrational numbers, like Pi, so long as we call Pi
         | '3.14'."
         | 
         | Pi is _not_ 3.14, so any mapping system that depends on you
         | cutting off your precision at some finite number of digits is
         | worthless.
        
           | phkahler wrote:
           | >> Your method is not a 1:1 mapping of reals to integers,
           | because you can't even define the position of a single
           | irrational number with your method. You don't know the final
           | digit, so you don't know the leading digit in your system.
           | You have absolutely no idea the ordering of even a single
           | irrational number in your system.
           | 
           | That is my point exactly. Why do we accept that there are
           | real numbers that cannot be defined by a finite string of
           | symbols, but there are no integers of that nature.
           | 
           | Getting back to the axiom of choice thing. How do you pick a
           | specific item from one of those ill-defined sets that include
           | numbers that don't have finite descriptions? How do you
           | choose something you can't name? It's absurdity all the way
           | down. ;-)
        
             | WastingMyTime89 wrote:
             | > Why do we accept that there are real numbers that cannot
             | be defined by a finite string of symbols
             | 
             | We don't accept it. It's easy to prove that it's the case.
             | 
             | I think you are unfamiliar with how R is defined. The
             | traditional definition of R is a completion of Q with
             | respect to the metric |x-y| (getting all Cauchy sequences
             | to converge). That's larger than infinite sequence of
             | decimals.
        
             | SamBam wrote:
             | That did not seem in any way to be your point. You said
             | 
             | > I've never been convinced that there are more reals than
             | integers
             | 
             | I'm explaining that your method of mapping reals to
             | integers is flawed, and so does not prove that reals and
             | integers have the same cardinality.
        
         | jacobmartin wrote:
         | Where is 1/3? Is it before or after (pi - 3)?
        
           | phkahler wrote:
           | We need to switch to the scheme that converts
           | name/description to an integer for that. If you mean the
           | repeating decimal, I don't know if it's before or after pi
           | because I can't identify (name) their locations.
           | 
           | It's the same argument. If I can accept that there are more
           | reals than reals with finite descriptions, why can't there be
           | integers without finite descriptions/positions?
        
             | jacobmartin wrote:
             | > If I can accept that there are more reals than reals with
             | finite descriptions, why can't there be integers without
             | finite descriptions/positions?
             | 
             | Because the integers are _defined_ as the finite successors
             | to the number 0 and their additive opposites. See, for
             | instance, the construction section of:
             | https://en.m.wikipedia.org/wiki/Integer
             | 
             | You can start talking about numbers that can't be
             | constructed in this way, but then you've ceased to be
             | talking about the integers.
        
               | phkahler wrote:
               | Is the size of the set of integers an integer?
        
               | jacobmartin wrote:
               | No, usually the cardinalities of infinite sets are
               | represented by things called aleph numbers:
               | https://en.m.wikipedia.org/wiki/Aleph_number.
               | 
               | But it isn't, in any case, an integer.
        
         | kjaa wrote:
         | I believe you are confusing rational for real numbers. Rational
         | numbers really do map to the integers, but I believe I can
         | "break" any purported isomorphism from reals to ints, by taking
         | two adjacent ints and asking what number the midpoint between
         | the corresponding reals map to.
        
           | phkahler wrote:
           | Please be specific. I never said the mapping has nice
           | properties regarding the ordering.
           | 
           | BTW I know rational numbers map to integers, and hence any
           | length tuple can be mapped to integers by pairwise combining
           | them in similar ways. But if any length tuple of integers can
           | map to a single integer, why can't an infinitely long set of
           | integers map to an integer? Like the string of digits in an
           | irrational number represented as a decimal?
        
             | Warwolt wrote:
             | The problem with mapping the reals to the integers is that
             | a list of real numbers is infinite in "two directions".
             | Each real can be potentially described with an infinite
             | number of digits, and you can have an infinite list of real
             | numbers (i.e. infinite sequences of digits). This is, I
             | believe, the basis for Cantor's diagonal argument, which is
             | a quite beautiful proof of there being more reals than
             | integers.
             | 
             | https://en.wikipedia.org/wiki/Cantor%27s_diagonal_argument
        
         | boole1854 wrote:
         | > Let's talk about real numbers between. 0 and 1 (because we
         | can map all of them to this range anyway). I can (count)
         | enumerate all of them. Simply flip the digits over the decimal
         | point, so 0.14159 become integer 95141.
         | 
         | Every integer has a finite number of numerals. Proof: by
         | definition, each natural number is either 0 or a successor to
         | 0. 0 is finite. And if _n_ is a finite length, then _n + 1_ is
         | a finite length. So by mathematical induction, all natural
         | numbers have finite length. A similar argument covers the
         | negative numbers, so all integers have finite lengths.
         | 
         | Since the decimal expansion of certain rational numbers are
         | infinite (e.g., 1/3 = 0.333...), the strategy of mapping each
         | number between 0 to 1 to an integer by flipping digits over the
         | decimal point does not work for all rationals, let alone all
         | reals.
        
       | jinpa_zangpo wrote:
       | I believe the definition of this set is contradictory. The set is
       | defined as all numbers that cannot be described with a finite
       | number of symbols. But that set must have a member whose value is
       | closest to zero. So you can describe at least one member of this
       | supposedly indescribable set.
        
         | less_less wrote:
         | It would at least require some careful setup to be coherent.
         | Like you won't be able to define "the least positive integer
         | that can't be uniquely described in fewer than fifteen words".
         | Basically your language won't be consistent if it allows you to
         | talk about things that can / can't be uniquely described in the
         | language itself.
         | 
         | My disagreement about this post is: I don't see what it has to
         | do with the axiom of choice. The only "choice" used here is
         | that this nonempty set of non-describable numbers (if indeed we
         | can define it at all) contains at least one element. That's a
         | tautology and doesn't require choice.
         | 
         | Also, the author may be interested in Skolem's paradox: in
         | particular, if the theory of the real numbers is consistent,
         | then there must be a construction with only countably many
         | reals, and so if I understand correctly, they would all be
         | describable.
        
         | johnnny wrote:
         | > But that set must have a member whose value is closest to
         | zero
         | 
         | This is not true for real numbers with the usual "less-than"
         | relation. In fact, considering real numbers, no open subset of
         | R admits a lower member.
         | 
         | According to Zorn's lemma, there exist some order relation
         | where your assertion is true, but I seem to remember from my
         | math course a few (!) years ago that Zorn's lemma is equivalent
         | to the axiom of choice.
        
           | ablob wrote:
           | This doesn't work for rational numbers either. Take any
           | rational a/b, then 0 < a/(2b) < a/b is both closer to zero,
           | and it is also a rational number.
        
           | WastingMyTime89 wrote:
           | According to the well-ordering theorem which is equivalent to
           | the axiom of choice (and Zorn's lemma), you can well-order
           | any set. That's usually when people start doubting the axiom
           | of choice because, well, that's quite unintuitive. There is a
           | joke about it which has already been posted here.
        
             | BeetleB wrote:
             | You can well order any set, but you need to have the
             | "right" ordering to do so. As your parent is pointing out,
             | the usual ordering on the reals is _not_ a well ordering,
             | and one cannot achieve a well ordering of the reals using
             | that ordering.
        
         | Slow_Dog wrote:
         | "the member whose value is closest to zero" is described by
         | those 14 symbols within the speech marks. As you say, the
         | definition is contradictory.
        
           | tsimionescu wrote:
           | Except no such member need exist. For example, there is no
           | number that corresponds to the description "the smallest
           | positive real number greater than 0".
           | 
           | Edit: note that even "the smallest positive _rational_ number
           | greater than 0 " doesn't exist, even though the rationals are
           | merely countably infinite.
        
             | tuatoru wrote:
             | This (no smallest positive element of Q) is sensible. But
             | how can you have a procedure that requires making an
             | infinite number of choices?[1] It's not an algorithm; they
             | have to terminate.
             | 
             | 1. Wikipedia explanation of the axiom of choice.
        
               | tsimionescu wrote:
               | I was only pointing out the problem with your argument
               | not arguing that the AoC is necessary or intuitive.
               | 
               | Also, algorithms that don't terminate are still
               | algorithms - especially if they keep emitting pieces of a
               | solution, instead of producing the whole solution at the
               | end.
               | 
               | For example, here is an algorithm for printing all of the
               | natural numbers:                 i <- 0       while true:
               | print(i)          i <- successor(i)
               | 
               | Finally, computability is not typically considered
               | necessary for a mathematical construct to exist or be
               | useful. Just like geometry sometimes studies shapes that
               | can't be constructed in the physical world, other kinds
               | of mathematics sometimes studies concepts that can't be
               | computed (at least not given known models of
               | computation).
        
         | scythe wrote:
         | The definition of this set _is_ contradictory, but not for the
         | reason you 've suggested. Rather, it is impossible to construct
         | the set of numbers which _can_ be defined by a finite string of
         | symbols, due to Richard 's paradox:
         | 
         | http://en.wikipedia.org/wiki/Richard's_paradox
         | 
         | Hence, it is also impossible to construct the complement of
         | this set. In fact, Richard's "diagonal argument" chooses
         | precisely the sort of number we ought not to be able to choose,
         | according to the blog post (but again, this just shows the set
         | is not well-defined).
        
         | hummusandsushi wrote:
         | I believe you are mistaken. This set of numbers is just the set
         | of reals minus the countable infinity of finitely representable
         | numbers. This set should still be densely ordered and so for
         | any member that could be chosen from this set, there is an
         | infinite number of elements closer to zero.
        
         | randallsquared wrote:
         | > _But that set must have a member whose value is closest to
         | zero._
         | 
         | Must it? Is there a real number closest to zero?
        
         | xen0 wrote:
         | _> But that set must have a member whose value is closest to
         | zero._
         | 
         | Must it?
         | 
         | "Let x be the smallest element of the half-open interval (0,
         | 1]" isn't really something that you can say *.
         | 
         | * Assuming the standard ordering of reals; sure you can say
         | it's defined for some well-order, but then you require the
         | Axiom of Choice anyway.
        
       | xen0 wrote:
       | The OP's argument seems to be just that you cannot always
       | construct a choice function.
       | 
       | But the entire point of the axiom of choice is to say "that's
       | fine; you're still allowed to use 'unconstructed' choice
       | functions in your proofs".
       | 
       | Would the OP reject the statement "let x be an element of the set
       | of numbers that cannot be defined with finitely many symbols";
       | such a statement does not depend on the AoC, but I think the OP
       | complains about making even a single choice.
        
         | lisper wrote:
         | > The OP's argument seems to be just that you cannot always
         | construct a choice function.
         | 
         | No, the argument is that _if you cannot describe the choice_
         | then you have a problem. My argument does not assume anything
         | about the choice function, it 's merely an informal observation
         | about its output.
        
           | xen0 wrote:
           | So the statement "let x be an element of y" is already
           | problematic?
           | 
           | If so, your problem is not with the axiom of choice.
           | 
           | Edit: "Not being able to describe the choice a choice
           | function makes" and "not being able to construct it" are
           | really the same thing.
        
         | rini17 wrote:
         | How is that different from claiming, say, there's an integer
         | betwen three and four, oh we can't construct it but that's not
         | a problem, gonna use it in the proofs anyway?
        
           | xigoi wrote:
           | An axiom has to be consistent with other axioms (that is,
           | adding it doesn't introduce a contradiction).
        
           | xen0 wrote:
           | That would let me derive your mother's phone number.
           | https://xkcd.com/704/
           | 
           | It's different because there's nothing that says such a
           | choice function _cannot_ exist (short of an axiom that says
           | explicitly so).
        
           | jbnicolai wrote:
           | That's essentially what axioms are. Wether it's reasonable or
           | not depends on your intuition of both axiom & implications.
           | 
           | This is where the axiom of choice is interesting; many people
           | feel that it seems intuitively reasonable, but the
           | implications are counterintuitive (e.g. banach-tarski)
        
           | contravariant wrote:
           | Well an integer between three and four is going to cause some
           | contradictions, depending on your definition of 'integer'.
           | 
           | You _can_ assume there exists a number between 3 and 4
           | though, no problems there.
        
           | jacobmartin wrote:
           | This is an interesting question that gets to the heart of the
           | constructivism debate, which I won't touch on here.
           | 
           | As for the integer example, the integers are all easily
           | (finitely) constructed (successors to 0 and their additive
           | opposites), so it isn't a consistent concept to say "This is
           | an unconstructable integer." Sets, on the other hand, are not
           | necessarily always constructable, unless you only take the
           | constructivist axioms.
        
       | wizeman wrote:
       | Perhaps we should start admitting to ourselves that infinity is
       | not really something that actually exists but instead it's just
       | an imaginary concept that we created but that has no
       | correspondence to anything that makes sense in our universe?
       | 
       | Or at least, some versions of infinity (I will leave which ones,
       | exactly, to the experts).
       | 
       | Hence all the paradoxes that derive from it.
       | 
       | I mean, perhaps we can work with _some_ version(s) of infinity,
       | without creating a plethora of paradoxes.
       | 
       | The natural numbers, for example, might be something that is
       | infinite and makes sense. However, perhaps talking about a
       | _single set_ that contains an infinite amount of natural numbers
       | might not make sense (or perhaps it still might?). Or perhaps we
       | just need a new language for talking about infinite things
       | without re-using the language we use for describing (finite)
       | sets, so that we don 't try to reuse concepts from finite objects
       | with infinite objects.
       | 
       | Or perhaps the only objects that exist (and can be meaningfully
       | talked about) are the computable ones. And/or perhaps "computable
       | function" is one that can use infinite amounts of time but only
       | finite amounts of storage (rather than infinite amounts of
       | storage).
       | 
       | Would it make sense to go back to classical finitism and re-
       | evaluate some of the choices that were made along the way to
       | where we are now?
       | 
       | I can't help but think about the many paradoxes that arise from
       | the very (supposed) existence of infinity.
       | 
       | For example, everybody thinks the halting problem is undecidable.
       | The halting problem is clearly not undecidable on finite-state
       | machines (i.e. real computers), it's only undecidable if you use
       | imaginary Turing machines (which supposedly can have literally
       | infinite state) as a model. But of course, Turing machines cannot
       | exist in our universe.
       | 
       | So it's rather ironic that people say the Halting problem is
       | undecidable, when this is only true for (impossible to construct)
       | imaginary machines but it's not true for the real machines that
       | we actually have and that are possible to construct.
       | 
       | Or think about Hilbert's paradox of the Grand Hotel.
       | 
       | Or the Banach-Tarski paradox.
       | 
       | Or Galileo's paradox.
       | 
       | Or other such paradoxes that only exist if you believe that
       | imaginary infinite objects could possibly exist.
       | 
       | Most people will tell you "well, it's just that our intuition
       | doesn't work for infinite things", but in this specific instance,
       | it's really hard for me to believe that it's our intuitions that
       | are mistaken rather than that we actually made some conceptual
       | mistakes along the way. To me, this argument sounds more like
       | gaslighting rather than a real convincing argument.
       | 
       | I'm not saying our intuition is always correct, though. For
       | example, the quantum properties of our universe are clearly not
       | intuitive but it's hard to argue that our universe is not
       | quantum, after all the scientific experiments that have clearly
       | verified this to be true.
       | 
       | I don't think that invalidates my argument, though, because the
       | convincing argument in that case is not "sorry, you must believe
       | my theory that the universe is quantum" despite not existing any
       | evidence that this was true, but rather, "sorry, but here's all
       | this real evidence that our universe truly is quantum".
       | 
       | Am I crazy here?
        
         | tsimionescu wrote:
         | The thing is, infinity is extremely useful for taking things to
         | the extreme. The halting problem is a very good example: as you
         | rightly point out, the halting problem is trivially decidable
         | for any finite Turing machine. However, the core of the
         | argument remains true, we just need to complicate it
         | significantly if we want to remain within the realm of finite
         | machines: instead of saying "there is no TM that can decide if
         | any arbitrary TM halts", we need to say something like "there
         | is no finite TM that can decide if any TM smaller than itself
         | halts in an amount of time less than that TM". They are
         | equivalent concepts on some level, but the second one is
         | significantly more complex, and harder to prove (assuming you
         | don't appeal to the infinite version, from which it easily
         | follows).
         | 
         | That is, even if you seek to do maths on finite but unbounded
         | sets, you will find mostly the same properties that infinite
         | sets have, but you'll have a much harder time working with the
         | concepts. It's true that there are some problems that infinite
         | sets have that unbounded finite sets don't, but those may well
         | be a worthy price to pay.
         | 
         | And either way, regardless of intuition, the logic is valid.
         | The Banach-Tarski "paradox" is perfectly logically valid,
         | though unintuitive. So, even if you could say that it's useless
         | and a result that we should avoid having in our theories, you
         | can't say that it's wrong, since it doesn't violate any rule of
         | logic.
        
           | wizeman wrote:
           | I can't really give a substantial response to your first
           | argument, as I'm not certain about all the implications that
           | you mention.
           | 
           | Your second argument, though, doesn't sound very convincing
           | to me.
           | 
           | There are many possible consistent logical frameworks
           | (perhaps an infinite amount of them? hah!) that are clearly
           | not adequate for describing our universe. Or at least, they
           | wouldn't make sense for us.
           | 
           | For a trivial (and perhaps stupid) example, consider an
           | alternate universe where we decided to use a logical
           | framework which can represent natural numbers, but where the
           | digit "2" is represented as "9" and the digit "9" is
           | represented as "7", etc. Let's say you have a 1-to-1 mapping
           | from one digit to another, compared to our usual
           | representation.
           | 
           | Or perhaps the mapping is a lot more complicated than that,
           | and not just about digits, but about entire numbers.
           | 
           | Let's say we decided to do that, for no good reason. Or maybe
           | there was a good reason at the time, but since then we have
           | figured out that it doesn't make sense anymore.
           | 
           | Yes, you could do exactly the same math in this logical
           | framework as we do in our usual ones, and no contradictions
           | would arise.
           | 
           | However, would that really be a wise idea? Wouldn't that just
           | lead to making unnecessary mistakes, sometimes even
           | conceptual ones, when considering that we would be prone to
           | confusing numbers in this system with the numbers that we use
           | normally?
           | 
           | I think language is also important here, not just logical
           | soundness. Especially if we want our intuitions to be
           | helpful.
           | 
           | Just to bring this back to something less abstract again, I
           | will just mention the following:
           | 
           | > However, the core of the argument remains true, we just
           | need to complicate it significantly if we want to remain
           | within the realm of finite machines: instead of saying "there
           | is no TM that can decide if any arbitrary TM halts", we need
           | to say something like "there is no finite TM that can decide
           | if any TM smaller than itself halts in an amount of time less
           | than that TM".
           | 
           | Well, I don't believe the core of the Halting problem to be
           | true, and that's exactly the language problem that I'm
           | talking about: confusing language leads us to making
           | conceptual mistakes.
           | 
           | For example, time is not necessarily relevant here.
           | 
           | I could argue that what the Halting problem really
           | demonstrates is that to analyze whether a finite-state
           | machine (FSM) halts, you need to use an FSM that can have
           | more state than the machine being analyzed (which is why it
           | stops working if you believe you can have infinite state).
           | 
           | This FSM with more state would always be able to decide
           | whether the other FSM halts in finite time, regardless of
           | whether the other machine halts or whether it never does.
           | 
           | In fact, that's exactly what cycle-detection algorithms do,
           | such as the tortoise and hare algorithm.
        
             | tsimionescu wrote:
             | The biggest problem with finite sets is that most
             | operations don't work nicely in finite sets.
             | 
             | For example, if we take the naturals to be a finite set,
             | and arbitrarily define some number G that is the largest
             | number, then common operations stop being
             | commutative/associative/distributive (e.g. x-y+z is no
             | longer equal to x+z-y, if x>G-z; or xx(y-z) [?] xxy - yxz).
             | Lots of simple equations stop having solutions at all. The
             | finite rationals also get similar problems. Your models
             | also have all these arbitrary parameters - the largest
             | positive integer, the largest negative integer, the
             | smallest fraction, the largest allowed amount of non-
             | repeating digits in a decimal representation of a fraction
             | - things like these become independent parameters, and can
             | be arbitrarily chosen to be different.
             | 
             | > Well, I don't believe the core of the Halting problem to
             | be true, and that's exactly the language problem that I'm
             | talking about: confusing language leads us to making
             | conceptual mistakes.
             | 
             | But it is - you still can't, in practice, construct a
             | useful program that depends on solving the "finite halting
             | problem", since you will need too much memory/time for
             | anything but the most trivial of examples. If you don't
             | beleive this, than it should be doable for you to create a
             | program that checks whether P=NP if we limit it to programs
             | that can run on, say, an 8086 processor and only use 16 KB
             | of RAM, right?
        
               | wizeman wrote:
               | > The biggest problem with finite sets is that most
               | operations don't work nicely in finite sets.
               | 
               | I didn't necessarily mean to suggest that we should not
               | work with infinite sets, only that perhaps we should not
               | call them "sets" (so as to not confuse them with the
               | finite ones), but rather something else, like "domains"
               | or whatever.
               | 
               | So perhaps you could say that those
               | commutative/associative operations are valid over the
               | domain of natural numbers. But you couldn't say, "X"
               | represents the cardinality of the set of natural numbers,
               | because there's no such thing as the set of natural
               | numbers.
               | 
               | Or, maybe the set of infinite natural numbers and
               | infinite rationals makes sense but the set of real
               | numbers doesn't (as they have different infinite
               | properties?).
               | 
               | I don't know exactly what would be a better solution
               | here, as I'm not a mathematician. I'm only trying to
               | suggest that there might be a better solution than what
               | we have right now, even if it would simply amount to just
               | using different language and not necessarily changing the
               | logical properties of the theory, so as to not arrive at
               | confusing conclusions.
               | 
               | > But it is - you still can't, in practice, construct a
               | useful program that depends on solving the "finite
               | halting problem", since you will need too much
               | memory/time for anything but the most trivial of
               | examples.
               | 
               | But the decidability of the Halting problem is not
               | actually a question of how long it takes for an algorithm
               | to solve it, is it?
               | 
               | The decidability of a problem is a matter of whether it's
               | actually possible _at all_ to construct an algorithm that
               | solves it, no matter how long the algorithm takes to
               | solve it.
               | 
               | That is, whether this algorithm is computable and
               | finishes in a _finite amount of steps_.
               | 
               | So I will say again that yes, the Halting problem is
               | decidable, as long as you don't use an absurd model to
               | answer the question (i.e. a model that does not represent
               | anything that can possibly be built in our universe even
               | in theory?).
               | 
               | The tortoise and hare algorithm solves the Halting
               | problem for FSMs in a finite amount of time and it only
               | needs twice the amount of storage as the underlying FSM
               | that is being analyzed.
               | 
               | > If you don't beleive this, than it should be doable for
               | you to create a program that checks whether P=NP if we
               | limit it to programs that can run on, say, an 8086
               | processor and only use 16 KB of RAM, right?
               | 
               | The P=NP is a problem regarding the resources required
               | during computation to solve a given problem, as far as I
               | understand.
               | 
               | I am not entirely sure how exactly this relates to the
               | decidability of the Halting problem when applied to
               | finite-state machines, though.
               | 
               | I mean, it's clear that the currently-known algorithms
               | that solve the Halting problem on FSMs, such as the
               | tortoise and hare algorithm, in the worst case take a
               | time to solve that is proportional to the cycle length of
               | non-halting FSMs.
               | 
               | But also note that these currently-known cycle detection
               | algorithms don't even try to analyze the FSM to decide
               | whether it halts, except by testing their states for
               | equality. This is due to the inherent definition of the
               | cycle detection problem.
               | 
               | So they can definitely be made more efficient, perhaps
               | for a large set of useful FSMs, even, as long as they
               | would be allowed to inspect the FSM's state transition
               | table. I'm not sure how efficient they could be, in
               | theory.
               | 
               | I would be cautious in taking any computational
               | complexity theory results at face value when we know that
               | they use Turing machines as models, which can give you
               | results that are the opposite of the results that apply
               | to FSMs. But I'm not an expert and admittedly, I'm not
               | entirely sure of what I'm talking about here.
               | 
               | Regardless, even if you can trust the computational
               | complexity results, to me this is an entirely different
               | question than the decidability of the Halting problem,
               | which is a "yes/no" problem.
        
         | housecarpenter wrote:
         | I agree that infinity doesn't exist in the universe, but this
         | isn't a problem for mathematics. Mathematics isn't about what
         | exists in the universe, it's about what exists in the realm of
         | concepts. Some concepts are more interesting than others, sure.
         | Cantor's work made it clear that infinite sets are pretty
         | interesting.
        
           | wizeman wrote:
           | The Flying Spaghetti Monster is also an interesting concept.
           | 
           | It can even be useful as a concept, in certain
           | discussions/contexts.
           | 
           | Should we take the Flying Spaghetti Monster as a core axiom
           | of our theories of physics?
           | 
           | Wouldn't this lead to us logically concluding for example,
           | that there has to exist something else beyond the universe
           | that we know about, even if this is not really true?
           | 
           | I mean, I know that I'm exaggerating and that the analogy is
           | not necessarily very good.
           | 
           | But I'm also not entirely sure how different is the Flying
           | Spaghetti Monster from the infinite objects that
           | mathematicians talk about and that lead us to logically
           | arrive at certain conclusions (that I would argue might not
           | really be true, in terms of things we can understand).
           | 
           | I'm not saying that I'm right and that most mathematicians
           | are wrong, necessarily. Perhaps it's just a linguistic issue,
           | I don't know.
        
       | dang wrote:
       | The thread this post is continuing:
       | https://news.ycombinator.com/item?id=34484011
        
       | boole1854 wrote:
       | The argument here seems to rely on a natural language intuition,
       | or perhaps even a physical or computational intuition, about what
       | it mean to "choose" something from a collection.
       | 
       | However, remember that the axiom of choice is technically a
       | statement about the _existence of certain functions_ ,
       | specifically functions that map from any set of non-empty sets to
       | elements of those sets. Whether or not the existence of those
       | functions says something about the possibility of "choosing" is
       | more of a terminological question about how to connect informal
       | natural language to mathematics than it is a substantive
       | mathematical question.
       | 
       | Turning to the mathematical question itself, it seems to me that
       | the author's intuitions fare poorly. The author argues that we
       | apparently cannot "choose" an element from a set that contains
       | only numbers that are not finitely describable. But the
       | mathematical question is about the existence of a mapping
       | function, and it seems that whether or not the numbers in each
       | set are finitely describable is irrelevant to this question:
       | 
       | Consider the function _f(x) = x + 1_. This function is defined
       | for all real numbers _x_. It maps each real number to another
       | real number. Note that the domain of the function includes even
       | those real numbers that are not finitely describable. The meaning
       | of the function itself is nevertheless clear and the function is
       | easy to write down. The fact that the domain of the function
       | includes indescribable numbers is not relevant to the question of
       | whether such a function exists mathematically.
        
         | Retric wrote:
         | f(x) = x + 1 is insufficiently defined here.
         | 
         | It's easy to pretend math is some universal language but it's
         | really several mutually exclusive languages.
         | 
         | Infinity is not a number. You can pretend it is and there are
         | several ways to build a mathematical system around such a
         | choice, but it isn't inherently correct. So in your example
         | f(x) may no longer include negative infinity and there is no
         | paradox, it may include multiple infinite numbers and the
         | lowest of them are now excluded, or you can say x and f(x) are
         | the same etc.
         | 
         | In the end there is no deeper truth here just the results of
         | various completely arbitrary definitions.
        
           | pdonis wrote:
           | _> Infinity is not a number._
           | 
           | It's not a real number, true. But that has no bearing at all
           | on the validity of f(x) = x + 1, since that function is well-
           | defined with both its domain and its range as all real
           | numbers and no others. You don't need infinity to be a number
           | to define that function.
        
           | jjgreen wrote:
           | _Infinity is not a number_
           | 
           | It depends on what you think a number is. Most people who
           | think about it agree a number is the name that we give to
           | equinumerate sets, so two eggs and two ducks are
           | equinumerate, we can give a bijection between these sets;
           | they have the same property, "twoness" and we abstract
           | twoness to the number 2. If so, then infinity is a number, it
           | this the number-ness of the set {1, 2, 3, ... }
        
             | quchen wrote:
             | > It depends on what you think a number is.
             | 
             | Let's not mix up teminology and semantics. What a number
             | here is is quite clear, we're all talking about relatively
             | standard math, the standard axiom of choice, and not some
             | opinion on numbers. Sure, you can also say a planet is a
             | mammal >>depending on what you think a mammal is<<, but
             | that doesn't lead to any sort of fruitful discussion.
        
               | jjgreen wrote:
               | So what would you say a number is if not a name for the
               | equivalences of set bijections? Genuinely interested,
               | what is 2?
        
             | vcxy wrote:
             | There is no bijection between the natural numbers and the
             | power set of the natural numbers. When talking about
             | bijections to study the sizes of sets, there is clearly not
             | just one infinity. Also consider the extended real line,
             | this makes infinity a perfectly good number as well in a
             | different way.
             | 
             | My point is that infinity is certainly not an individual
             | number. It represents quite a few concepts, all of which
             | have some number like qualities.
        
               | jjgreen wrote:
               | Indeed, but when people say "infinity is not a number"
               | they generally mean 0, and that is (in my view) as much a
               | (cardinal) number as 2. Once you have that established,
               | then you go on to discuss the higher s and the continuum
               | hypothesis ...
        
               | vcxy wrote:
               | > when people say "infinity is not a number" they
               | generally mean aleph-0
               | 
               | Sure, maybe. I think saying "infinity is a number" is
               | wrong regardless. I'm happy with aleph-0 being a number,
               | and I'm happy with it having a label of infinity. I'm
               | just not happy with treating infinity as a single thing.
               | We're totally on the same page in every way that matters
               | though. I'm also certainly with you on questioning
               | Retric's comment for a number of reasons, haha.
               | 
               | edit: Actually...I'm not so sure they mean aleph-0. I
               | think frequently they mean the infinity of the extended
               | real line. Maybe it depends on the context of the
               | conversation.
        
               | jjgreen wrote:
               | It's actually a great ice-breaker for first-year maths
               | undergrads, when talking about limits or whatever drop in
               | "... to infinity and beyond!", laughter follows, then say
               | "You think I'm joking? Put down your pens ..." and
               | informally outline the aleph hierarchy, then things get
               | rather quiet.
        
               | vcxy wrote:
               | Ahh I wish I had gotten the chance to interact with new
               | math majors more often. In grad school most of my
               | students were engineering majors. I'm not super likely to
               | end up back in academia. My published research
               | is...sparse. I have post-doc opportunity but it's not the
               | greatest, and the route to life stability is not so
               | clear. Are you a math professor?
        
               | jjgreen wrote:
               | No, a developer; I spent several years as applied maths
               | researcher though, and UK universities love to use
               | researchers as cheap lecturers (not that I'm complaining,
               | rather enjoyed it).
        
               | vcxy wrote:
               | Cool. I have a second interview for a software developer
               | position at a quantum computing place this week.
               | Hopefully I'll be following you into the field!
        
           | syzarian wrote:
           | Your post doesn't make sense mathematically. As a function
           | from the reals to the reals the function f(x) = x+1 doesn't
           | "include negative infinity" (have no idea what that is
           | supposed to mean). Negative infinity is not a real number and
           | is not in the domain of the function or its range. If you
           | think "infinity" (which one?) is not a number then do you
           | believe there are no infinite ordinals? How do you justify
           | the view that countably infinite ordinals don't exist?
        
             | Retric wrote:
             | You are making assumptions, define the lowest real number.
        
               | syzarian wrote:
               | There are well defined models of the real numbers. You
               | don't know enough mathematics to comment intelligently on
               | this topic. What you wrote above makes no sense.
               | 
               | EDIT: There is no least real number under the standard
               | ordering. There is a least real number for a given well
               | ordering. If you are asking me to define a well ordering
               | on the reals then your request question should have been
               | stated differently.
        
               | Retric wrote:
               | You said models not model here, so you acknowledge there
               | isn't a single universal definition.
               | 
               | I've seen some truly bizarre definitions of the reals
               | which are internally consistent and include things like
               | the lowest possible number.
        
               | syzarian wrote:
               | You don't know enough set theory and model theory. Of
               | course there are different models of the real numbers.
               | There are non isomorphic models of the first 4 axioms of
               | Euclidean geometry even. A given set of axioms can have
               | lots of different models.
               | 
               | The widely accepted first order axioms of the natural
               | numbers have nonstandard models. Of course each such
               | model has an isomorphic initial segment. There are two
               | things going on. There is the set of axioms and there are
               | models for those axioms. I think in your usage: "define
               | real number" corresponds to "give the axioms for the real
               | numbers". I'm not sure though.
        
               | Retric wrote:
               | Non standard models of arithmetic really takes me back.
               | Anyway, axiom/models is the common terminology though I
               | recall a rather long rant that postulate is better once
               | you start talking about multiple incompatible systems.
        
               | messe wrote:
               | Please consider that the poster telling you that you do
               | not know enough to comment intelligently about this topic
               | may be correct.
               | 
               | > Anyway, axiom/models is the common terminology though I
               | recall a rather long rant that postulate is better once
               | you start talking about multiple incompatible systems.
               | 
               | Model has a very specific definition in this area of
               | mathematics. This sentence is nonsense.
        
               | Retric wrote:
               | I am describing an actual event with that sentence. Are
               | you suggesting that someone can't rant about such things?
        
               | Arkhaine_kupo wrote:
               | The negative reals are an infinite set, that doesn't make
               | "negative infinity" a real number though. Negativy
               | infinity is a concept, that can be used in some math
               | frameworks, but it is not defined within R.
               | 
               | See this exchange for details
               | 
               | https://math.stackexchange.com/questions/2812355/does-
               | infini...
        
               | Retric wrote:
               | I am trying to allude to transfinite numbers and annoy
               | mathematicians without losing a general audience, feel
               | free to ignore such things.
        
         | JackFr wrote:
         | I'm not sure you've shown anything. _f(x) = x + 1_ is a mapping
         | from the reals to the reals. A choice function, with respect to
         | the axiom of choice, is a mapping from the set to an element.
        
           | boole1854 wrote:
           | The point of my example is to show that the functions can
           | exist even if their domain includes "indescribable" numbers.
           | It shows that trivially the "indescribable-ness" of numbers
           | in the domain of the function is not relevant to whether the
           | function exists.
           | 
           | The author's argument relies on "indescribable-ness" somehow
           | blocking the ability to "choose" an element. But that trades
           | on a vague, intuitive understanding of "choosing", which
           | doesn't capture what the AC is about. The AC is about the
           | existence of certain functions. Since the "indescribable-
           | ness" of the domain of a function is not relevant to its
           | existence, the author's argument doesn't successfully attack
           | the AC.
        
             | lisper wrote:
             | > The point of my example is to show that the functions can
             | exist even if their domain includes "indescribable"
             | numbers.
             | 
             | But that is not at issue. What is at issue is whether or
             | not a function exists whose domain is an (infinite) _set_
             | of indescribable numbers and whose range is a member of the
             | set. The existence of such a function is far less clear.
             | 
             | Of course you can postulate the existence of such a
             | function. But you can also postulate the existence of an
             | oracle for the halting problem. My gut tells me that these
             | two ideas have a lot in common.
        
               | boole1854 wrote:
               | Remember that I am not here trying to argue for the truth
               | of the AC; I'm arguing against the author's argument. The
               | author's argument, as I read it, relies on an intuition
               | that "choosing" an element that is not-finitely-
               | describable is problematic. However, the mathematical
               | meaning of "choosing", in the context of the AC, is a
               | statement about the mathematical existence of a certain
               | function. To respond to the author's intuition requires
               | only showing that not-finitely-describable domains are
               | not a "problem" for the existence of functions. Maybe it
               | is difficult to "choose", in some natural language sense,
               | an input to f(x) = x + 1 such that the output x + 1 is a
               | real number (e.g., you can't write down such an input one
               | digit at a time on a piece of paper, even an infinitely
               | long piece of paper). But this does not mean it is
               | problematic for the function to exist in the mathematical
               | sense.
        
               | lisper wrote:
               | FYI, I am the author.
               | 
               | > Remember that I am not here trying to argue for the
               | truth of the AC; I'm arguing against the author's
               | argument.
               | 
               | Yep, I get that.
               | 
               | > The author's argument, as I read it, relies on an
               | intuition that "choosing" an element that is not-
               | finitely-describable is problematic.
               | 
               | Not quite. Choosing an element from a set where _all_ of
               | the members are not-finitely-describable is problematic.
               | 
               | > However, the mathematical meaning of "choosing", in the
               | context of the AC, is a statement about the mathematical
               | existence of a certain function.
               | 
               | Yep, I get that.
               | 
               | > To respond to the author's intuition requires only
               | showing that not-finitely-describable domains are not a
               | "problem" for the existence of functions.
               | 
               | No, you would have to show that defining a function on
               | _this particular set_ is not a problem. Note that this is
               | _not_ the same thing as defining a function on _elements_
               | of this set. That is obviously not a problem. But
               | defining a (non-trivial) function _on the set itself_ is.
        
               | riversflow wrote:
               | I just don't see what describability, or as jiggawatts
               | calls it, naming, has to do with AC. AC is built upon an
               | indexed set[1]. That way you can just reference the
               | index. It seems extremely contrived to me to argue that
               | being finitely nameable has any bearing on underlying
               | mathematical principle, nothing about any form of
               | infinity is relatable to the physical confines of this
               | universe, of which language is a member, but the concept
               | of infinite is very useful.
               | 
               | [1]https://en.wikipedia.org/wiki/Axiom_of_choice
        
               | syzarian wrote:
               | There are models of ZFC in which every real number is
               | definable. You may be interested in the top answer at
               | this mathoverflow.net question:
               | 
               | https://mathoverflow.net/questions/44102/is-the-analysis-
               | as-...
        
               | lisper wrote:
               | That is very interesting (have an upvote!) but I don't
               | see how it invalidates my argument. If every real is
               | definable then the set of non-definable reals is empty
               | and AoC does not apply at all.
        
               | syzarian wrote:
               | The notion of countable and not countable does have some
               | nuance. In the models that every real is definable that
               | model, from within, believes it is uncountable since
               | there is no bijection with a countable set from within
               | that model. However, from the outside that model is
               | countable.
               | 
               | The point is there is a lot of nuance and no matter what
               | one believes regarding choice there will be unintuitive
               | results. For instance, when the axiom of choice fails you
               | can have an infinite set with not countably infinite
               | subset. The reals can be a countable union of countable
               | sets. There is a set that can be partitioned into
               | strictly more equivalence classes than the original set
               | has elements, and a function whose domain is strictly
               | smaller than its range. In fact, this is the case in all
               | known models.
        
               | Kranar wrote:
               | We need to be careful with the language here. It's not
               | true that there are models of ZFC where the real numbers
               | are actually definable. What's true is there are models
               | of ZFC where what is considered a "real number" from
               | within that model is always definable (but will not be
               | definable within that model). However, those sets do not
               | form the actual real numbers, they are simply sets that
               | from within the model satisfy a given definition of the
               | reals without actually being the reals.
               | 
               | A similar result is that there are models of the reals in
               | ZFC that are countable, even though the reals are not
               | countable. How can this be? Well it's similar to the
               | above argument, those models do not represent the actual
               | real numbers.
               | 
               | All this means is that it's not possible to categorically
               | define real numbers in first order ZFC. Any axiomatic
               | definition of the reals in first order ZFC will have some
               | models that satisfy those axioms but are not actually
               | real numbers.
        
               | syzarian wrote:
               | I'm not a logician or set theorist. I studied commutative
               | algebra and, naturally, accept the axiom of choice. When
               | you say, _"the actual reals"_ I think you mean the
               | standard notion of a real number that we all are used to
               | using. But those models that we are talking about are
               | models of the axioms of the reals. So, they are the
               | actual real numbers as far as that model is concerned,
               | right?
        
               | Kranar wrote:
               | You got it, absolutely. So yes there are models where as
               | far as the model is concerned there is a set that
               | satisfies the axioms of real numbers, and furthermore all
               | elements of that set are actually definable (but not
               | definable from within that model), but that set is not
               | the standard notion of real numbers. It's just that it's
               | impossible to provide a set of axioms that defines the
               | real numbers and only the real numbers (in first order
               | logic), so for any definition of the real numbers you end
               | up with a model that will satisfy that definition but
               | doesn't actually model the "standard notion of real
               | numbers".
               | 
               | The reason I brought this up is because when you said
               | that there are models of ZFC where every real is
               | definable, those reals are only definable from outside of
               | the model. From within that model there are still
               | undefinable reals and hence I don't think it matters too
               | much to the point lisper is making.
               | 
               | Maybe to put it another way, there is no model of the
               | reals where within that model all reals are definable,
               | and hence lisper's point stands.
        
               | syzarian wrote:
               | Understood. I wasn't trying to address lisper's directly
               | as such. My poorly stated goal was to point out that
               | intuition regarding uncountable sets is shaky at best.
               | There are weird results regardless of one's views because
               | of nonstandard models.
        
               | cgio wrote:
               | Intuitively I would argue the definition of
               | indescribability is loaded with semantics that can
               | sprinkle paradoxes at will. A set from my perspective has
               | at least two interfaces, one to populate it and one to
               | query it. By making the population interface
               | "indescribable" you get an "indescribable" method to
               | query. Worst case you start from a describable superset
               | of the indescribable one and test the "indescribable "
               | set population method to get you an indescribable set
               | member. Which makes the indescribable actually
               | describable in terms of this process. "?" on this whole
               | statement, late night intuition from an indescribably bad
               | at math sleepy guy.
        
               | recursive wrote:
               | > one to populate it and one to query it
               | 
               | I'm not a math expert, but "populating" a set strikes me
               | as an imperative programming mindset. In math, I
               | understand a set more like a function returning a boolean
               | that indicates membership. If the contents of the set
               | change, then it's just a new set.
        
               | lisper wrote:
               | Yeah, this. A set does not have "two interfaces", it only
               | has one: a membership function that takes an object and
               | returns a boolean indicating if the object is a member of
               | the set or not. Note that this function need not be
               | computable, only well-defined.
        
               | loicd wrote:
               | The nature of the elements of the set does not matter,
               | since the existence of a choice function on the set A
               | guarantees the existence of a choice function on the set
               | B as soon as there is a bijection between A and B. Thus,
               | no matters how counter-intuitive or unnatural one finds
               | the elements of a set, or whether they model physical
               | reality, what matters for the axiom of choice is whether
               | one can construct a bijection with a set that has a known
               | choice function.
               | 
               | By the way, it is worth keeping in mind _how_ Godel
               | proved the consistency of the axiom of choice. Roughly
               | speaking, the steps are: start with a model of ZF, build
               | from it an inner model where all sets are definable (in a
               | sense) in terms of ordinals, that model (called the
               | "constructible universe") satisfies the axiom of choice.
               | In other words, the axiom of choice holds as soon as you
               | assume that _all_ sets are constructible.
        
             | 6gvONxR4sf7o wrote:
             | It seems like the validity of what you're saying is
             | intertwined with the axiom itself. You're going from a
             | comment about the domain (the domain includes indescribable
             | elements) to a comment about specific arguments (arguments
             | may be indescribable).
             | 
             | Ultimately, you're saying that if we have an indescribable
             | x, then x+1 is conditionally describable (to abuse
             | terminology). But I think that just sidesteps the question.
             | Even then, if we're taking a computability lens, then
             | indescribable(x) implies indescribable(x+1), which would
             | seem to imply that applying your function to indescribable
             | x's is a moot point.
             | 
             | The fact that your function is finitely describable even
             | though we can't describe each of its applications is
             | admittedly a mindfuck.
        
           | jvanderbot wrote:
           | Like f(x) = 5?
        
             | yaakov34 wrote:
             | Wouldn't work, since 5 is not a member of the set of
             | indescribable numbers. I am still not sure that the
             | original example is good, since serious difficulties with
             | the axiom of choice have to do with infinite collections of
             | sets, especially uncountably infinite, not with choosing a
             | member of a single difficult-to-describe set. But you can't
             | handle these non-describable numbers very easily.
        
               | [deleted]
        
               | davrosthedalek wrote:
               | That's fine. The point is that the x is also not in the
               | set of indescribable numbers (because you have described
               | it with f(x)=5), assuming f is describable.
        
             | davrosthedalek wrote:
             | The problem is that for every finite description of f, the
             | x that fulfills f(x)=5 is not in the set of numbers than
             | cannot be described by a finite set of symbols. (I assume
             | here that there is only one solution of f(x)=5)
        
         | kazinator wrote:
         | Let's pin it down!
         | 
         | To choose an element from every one of a set of sets, we must
         | describe the method of choice as a halting algorithm executable
         | by a Turing Machine.
        
         | ketzu wrote:
         | I wonder if the set of undescribable numbers is well defined at
         | all. It seems to run into the same problem as the "minimum un-
         | interesting number is interesting" as a function chosing a
         | number from that set would be a finite description of that
         | number, removing it from the set. (This already happens if you
         | could select a finite subset of those numbers, as finite
         | numbers could be ordered and you could just select the
         | minimum.)
         | 
         | edit: Hm... thinking about it. The finitely-many-symbols
         | argument also only holds if you consider finitely many
         | interpretations of those symbols and if there actually are only
         | finitely many symbols.
        
           | LegionMammal978 wrote:
           | It's not too mind-boggling: we can describe the set as a
           | whole without any issues, we just can't _uniquely_ describe
           | any particular number in the set. That doesn 't stop us from
           | generalizing over all numbers in the set, or even over all
           | numbers in a describable subset, such as "the set of all
           | undescribable primes".
        
             | ketzu wrote:
             | > we can describe the set as a whole without any issues
             | 
             | But it does not mean the set is well defined. The
             | description is not necessarily clear an unambigious. It
             | might not be consistent under the generalizations we come
             | up with.
             | 
             | > "the set of all undescribable primes"
             | 
             | Interesting example, because the set is trivially empty
             | under the assumption that we can describe any finite
             | natural number by writing down its decimal expansion.
             | (Based on that you can either argue that the primes are a
             | subset of the natural numbers or that the primes are
             | countable.)
             | 
             | I see no reason to believe that "undescribable" is a well
             | defined property so far.
        
           | resource0x wrote:
           | To me, the expression "The set of undescribable numbers" is
           | meaningless. While discussing plausible-sounding, but
           | meaningless, linguistic constructs like this, we descend to
           | the intellectual level of chatGPT. My conjecture is that one
           | can design a prompt to chatGPT such that the output would
           | become a decent PhD thesis with no internal contradictions,
           | but a complete BS otherwise.
        
           | librexpr wrote:
           | I think if we take "description of a number" to mean "ZF
           | formula that uniquely picks out that number", then that
           | cannot be defined, because a formula picks out a number when
           | it is true for that number and false for all others, but by
           | Tarski[0], the truth predicate cannot be defined inside the
           | logic itself. So "the set of all numbers which cannot be
           | described" cannot be talked about using ZF.
           | 
           | However, there is a way around it, by taking as axiom that ZF
           | is consistent, choosing some model M of ZF, and then talking
           | about the set S of numbers inside M that cannot be described.
           | 
           | [0] https://en.wikipedia.org/wiki/Tarski%27s_undefinability_t
           | heo...
        
             | actually_a_dog wrote:
             | Bingo. The issue here is that ZF and ZFC are both first
             | order theories. They can only talk about things that _can
             | be_ defined within them, not things that _cannot_ be
             | defined within them. The wiki article talks about that
             | where it says:
             | 
             | > Informally, the theorem says that the concept of truth of
             | first-order arithmetic statements cannot be defined by a
             | formula in first-order arithmetic. This implies a major
             | limitation on the scope of "self-representation". It is
             | possible to define a formula True(n){\displaystyle True(n)}
             | whose extension is T*,{\displaystyle T^{*},} but only by
             | drawing on a metalanguage whose expressive power goes
             | beyond that of L.L. For example, a truth predicate for
             | first-order arithmetic can be defined in second-order
             | arithmetic. However, this formula would only be able to
             | define a truth predicate for formulas in the original
             | language L.L. To define a truth predicate for the
             | metalanguage would require a still higher metametalanguage,
             | and so on.
        
           | mckeed wrote:
           | That's the point though, right? Without Choice, there need be
           | no way to select a number or subset from the set, so there's
           | no paradox.
        
           | planede wrote:
           | I think the resolution is along the lines of this:
           | 
           | * ZFC does not have a unique model that satisfies the axioms.
           | 
           | * Each of those models have different "undescribable numbers"
           | with different properties.
           | 
           | But it was a long time I touched model theory, and it was
           | just an optional extra course at uni.
           | 
           | https://en.wikipedia.org/wiki/Model_theory
        
           | hilbertseries wrote:
           | https://en.m.wikipedia.org/wiki/Computable_number
        
             | mannerheim wrote:
             | There are uncomputable numbers which can still be defined.
             | 
             | https://en.wikipedia.org/wiki/Chaitin%27s_constant
        
               | [deleted]
        
         | kybernetikos wrote:
         | > But the mathematical question is about the existence of a
         | mapping function, and it seems that whether or not the numbers
         | in each set are finitely describable is irrelevant to this
         | question:
         | 
         | If it's possible to construct the set at all, then there is by
         | definition no function that can be described in a finite number
         | of symbols that can select an element from the set (since if a
         | finitely describable function can define a member of the set,
         | it shouldn't be in the set - you'd define the element as being
         | the result of the function).
         | 
         | I don't know what the scholarly view is on relying on functions
         | that _cannot be defined in any human language, including
         | mathematics_. I would intuitively lean towards not allowing
         | them in proofs, but perhaps they 're useful mathematically?
         | 
         | Another interesting aspect - these are numbers, so presumably
         | there is a smallest one, but identifying that is definitionally
         | ruled out too, since 'smallest element in the set of numbers
         | indescribable in a finite sequence of symbols', is itself a
         | finite sequence of symbols.
        
           | hackandthink wrote:
           | "The well-ordering theorem together with Zorn's lemma are the
           | most important mathematical statements that are equivalent to
           | the axiom of choice"
           | 
           | This is all crazy stuff, but it would be consistent:
           | 
           | No axiom of choice -> no well-ordering -> no smallest number.
        
           | syzarian wrote:
           | Consider the interval (0, 1). This set of real numbers
           | contains no smallest element. It is not true that all sets of
           | real numbers have or should have a least element. It is a
           | property of the standard model of the nonzero integers that
           | all subsets have a least element.
           | 
           | It should be noted that there are models of ZFC in which each
           | real number is definable. You may be interested in reading
           | the top answer on this mathoverflow.net question:
           | 
           | https://mathoverflow.net/questions/44102/is-the-analysis-
           | as-...
        
             | [deleted]
        
       | huitzitziltzin wrote:
       | This part:
       | 
       | > Here's how: consider the set of numbers that cannot be
       | described using any finite collection of symbols. Such numbers
       | must exist *because there are only a countably infinite number of
       | numbers that can be described using a finite collection of
       | symbols*, but there are an uncountably infinite number of real
       | numbers.
       | 
       | That's clearly wrong. The real numbers between 0 and 1 (an
       | uncountable collection) are describable with a finite collection
       | of symbols - the digits between 0 and 9. Each real number is an
       | equivalence class of sequences with the same limit. (It's been a
       | while since I took real analysis but I believe this is the
       | construction due to Weierstrass?)
        
         | danbruc wrote:
         | The author means finite sequence over finite alphabet, i.e. the
         | finite collection of symbols does not refer to the alphabet.
        
           | [deleted]
        
         | johnnny wrote:
         | The set of numbers that can be written with a finite number of
         | 0-9 digits is countable. That set is called the decimal
         | numbers, and is a subset of the rational numbers which itself
         | is countable too.
        
           | huitzitziltzin wrote:
           | The set of numbers which can be written _with finitely many_
           | members of a finite set of symbols is countable, sure.
           | 
           | That doesn't sound like the claim the author is making in the
           | sentence I quoted but maybe I am being uncharitable?
        
             | johnnny wrote:
             | If you allow infinitely long descriptions, then I don't
             | know what's the score. In the case of "descriptions" that
             | amount to (potentially infinite) decimal expansions, I
             | think an argument akin to Cantor's diagonal will show that
             | you can't possibly describe all real numbers (or all
             | numbers between 0 and 1), but I'm not confident I can
             | assert this to be true.
        
       | renewiltord wrote:
       | This doesn't make sense. That's not what AoC means. It is a
       | statement that the cartesian product is non-empty if the sets are
       | non-empty.
       | 
       | One might as well say "Well, if I were to lobotomize you, you
       | would not be able to choose". The "choice" here is a term of art.
       | There is no actual "choosing" going on here.
        
       | Sankozi wrote:
       | "consider the set of numbers that cannot be described using any
       | finite collection of symbols" - isn't that set empty?
       | 
       | If "numbers" mean "real numbers" then I can describe easily
       | "smallest positive number in the set" and "biggest negative
       | number in the set" - so neither negative, 0 nor positive real
       | numbers might be in the set.
        
         | norrius wrote:
         | What is the "smallest positive number" in the set {x | x in
         | reals and 0 < x < 1}?
        
         | tsimionescu wrote:
         | It's trivial to prove that the number you are describing
         | doesn't exist:
         | 
         | Let x be the smallest positive real greater than 0. Since x >
         | 0, x/10 < x, and x/10 > 0. But now we have a contradiction,
         | since x was supposed to be the smallest positive real number.
         | So, x doesn't exist.
         | 
         | Edit: note that this doesn't even require real numbers. Even
         | for the rational numbers, with common definitions of less-than,
         | there is no smallest positive rational number >0, per the same
         | logic.
        
       | mjburgess wrote:
       | The axiom of choice is intuitive because its seems one should
       | always be able to take many aggregates of elements and make a new
       | aggregate.
       | 
       | The axiom of choice is counter-intuitive because it provides no
       | reason to, in the case of a uniform aggregate, choose one element
       | over another. Nor a reason to suppose in the case of a "complex
       | aggregate" that "choice" is even well-defined.
       | 
       | Perhaps one way of thinking of the issue here is the tension
       | between the discrete and continuous. Of course, one can always
       | recombine even inifnite number of discrete things.
       | 
       | But what does it mean to "choose" a "point" on a "line" ? Can one
       | "choose" pi?
       | 
       | One could, for example, take the view that the very notion of
       | membership, "choice", etc. is conceptually inadequate for
       | handling geometrical matters such as the reals.
        
       | orlp wrote:
       | > Such numbers must exist because there are only a countably
       | infinite number of numbers that can be described using a finite
       | collection of symbols, but there are an uncountably infinite
       | number of real numbers. So not only are there numbers that cannot
       | be described using a finite number of symbols, there are vastly
       | more of these than numbers that can be so described.
       | 
       | Why is almost no one more critical of this? A 'number' you can't
       | describe or do computation with? Why do we still call this a
       | number? If I ask Google to define number I get
       | 
       | > an arithmetical value, expressed by a word, symbol, or figure,
       | representing a particular quantity and used in counting and
       | making calculations.
       | 
       | Almost all (in the mathematical sense) 'real' 'numbers' can not
       | be expressed, nor can be used for counting or making
       | calculations.
       | 
       | Over the years I'm becoming more and more convinced we should
       | reject the 'real' numbers as our default mathematical object for
       | numbers, and switch to a basis that has a computable foundation.
       | There is some work being done in this area, but not nearly enough
       | in my opinion:
       | https://en.wikipedia.org/wiki/Computable_number#Use_in_place...
        
         | mik1998 wrote:
         | Real numbers are interesting and math is not about computation,
         | counting or calculation.
        
           | hgsgm wrote:
           | math is not _only_ about computation, counting or
           | calculation.
        
         | hgsgm wrote:
         | Mathematicians hijacked numbers.
         | 
         | Computer scientist don't accept noncomputable objects.
         | Mathematicians do.
         | 
         | To a scientist, existence means "you, or someone, can find it,
         | at least potentially". To a non-constructive mathematician (the
         | common kind) existence means "you can't prove it _doesn 't_
         | exist, and I choose to believe it does, at least in the current
         | game I am playing."
         | 
         | Mathematicians use the term "real numbers" to describe
         | precisely the set of numbers you get when you take all the
         | numbers that could plausibly be computed, _and then declare
         | that all the holes between numbers we can individually compute
         | /describe are completely full of numbers._ This is justified
         | because you can't prove that any specific one of them is
         | impossible to write.
         | 
         | (But there are some so called "real" numbers that are
         | impossible to calculate, even approximately, like Chaitin's
         | constant https://en.m.wikipedia.org/wiki/Chaitin%27s_constant
         | but you could write it in infinite time if God told you the
         | digits.)
        
           | drdeca wrote:
           | > Computer scientist don't accept noncomputable objects.
           | 
           | People who study computability certainly do, and I would
           | think they count as computer scientists.
        
           | shkkmo wrote:
           | > But there are some so called "real" numbers that are
           | impossible to calculate, even approximately, like Chaitin's
           | constant https://en.m.wikipedia.org/wiki/
           | 
           | Just because you can create a definition, doesn't mean that
           | definition names a thing. We can define "the set of all sets
           | that do not contain themselves", but such a definition is
           | nonsensical in that trying to apply it creates a
           | contradiction. To me, the incomputability of Chaitin's
           | constant seems very similar, except it's contradiction is
           | buried inside the halting problem.
        
           | bmitc wrote:
           | > Computer scientist don't accept noncomputable objects.
           | Mathematicians do.
           | 
           | Computer scientists don't have a choice. And not all
           | mathematicians do accept non-computable objects. Constructive
           | and intuitionistic mathematics are two approaches that bring
           | some questions (doubt?) of the continuum.
        
         | dia80 wrote:
         | You are going to love this:
         | 
         | How real are real numbers?
         | 
         | https://arxiv.org/abs/math/0411418
        
         | [deleted]
        
       | moron4hire wrote:
       | A variable name and the thing it points to are not the same
       | thing. Symbols aren't numbers. They are a means to communicate
       | numbers.
        
       | psychoslave wrote:
       | Naming is not a requirement to selecting. I can pick a tool, and
       | even use a tool, without having to name it. Most selection
       | process out there work without relying on any explicit name.
       | 
       | In the theory of natural selection for example, specimens are
       | selected without being named.
        
       | cousin_it wrote:
       | > _consider the set of numbers that cannot be described using any
       | finite collection of symbols. Such numbers must exist because
       | there are only a countably infinite number of numbers that can be
       | described using a finite collection of symbols, but there are an
       | uncountably infinite number of real numbers. So not only are
       | there numbers that cannot be described using a finite number of
       | symbols, there are vastly more of these than numbers that can be
       | so described._
       | 
       | Maybe you already know, but this argument is wrong, see Hamkins
       | "Pointwise definable models of set theory". It's a triple whammy:
       | 1) You can't state the argument formally, because the property of
       | describability is undefinable (Tarski's undefinability of truth).
       | 2) Whatever your theory claims about uncountability, from the
       | outside there's always a countable model of it (Lowenheim-
       | Skolem). 3) And if you have a countable model, there's nothing
       | stopping you from having a finite formula for every member of it
       | (making it a "pointwise definable model"). All the details are
       | worked out in the paper: yes Virginia, there are models of ZFC
       | whose every member - every set, every function, every real number
       | - is definable by a finite formula.
       | 
       | Also a lot of nice counterexamples in this reddit thread:
       | https://www.reddit.com/r/math/comments/d9sp9h/the_math_tea_a...
       | 
       | For example:
       | 
       | > _" Take your favorite real in some model M. We can take a
       | forcing extension to a model M[G] in which the real is coded in
       | the continuum function below aleph_omega. Now it's definable."_
       | 
       | Or:
       | 
       | > _" There is a computable real that ZFC doesn't prove is a
       | computable real."_
        
       | tel wrote:
       | AC fails to exist, in its normal form, in constructive or
       | intuitionistic formulations of math. This post appeals to that
       | sort of mathematics and thus correctly identifies that it
       | wouldn't make sense there.
       | 
       | There are weaker forms of AC which do exist, though. And
       | formulations of math where AC is necessary and obvious, but
       | doesn't really imply the same thing as classical AC. But these
       | formulations of math are very unlike the classical one we
       | normally enjoy.
       | 
       | For instance, they often disallow the construction of the Reals
       | altogether.
        
         | quchen wrote:
         | Why does it fail to exist? It's an axiom that can be added to
         | intuitionistic axiom systems just as well as for classical
         | ones, no? It's orthogonal to the law of the excluded middle
         | after all. In constructive mathematics with AC, you would just
         | call the selected set element AC(your-set).
        
           | tel wrote:
           | It's been a while so I've forgotten the details, but I
           | believe in some versions of constructive mathematics AC is
           | enough to derive LEM.
        
       | n4r9 wrote:
       | This has a similar feel to Russell's paradox:
       | 
       | Let R be the set of all sets that are not members of themselves.
       | If R is not a member of itself, then its definition entails that
       | it is a member of itself; if it is a member of itself, then it is
       | not a member of itself, since it is the set of all sets that are
       | not members of themselves.
       | 
       | https://en.m.wikipedia.org/wiki/Russell's_paradox
       | 
       | Also the Lowest Uninteresting Number paradox:
       | 
       | If there exists a non-empty set of uninteresting natural numbers,
       | there would be a smallest uninteresting number - but the smallest
       | uninteresting number is itself interesting because it is the
       | smallest uninteresting number,
       | 
       | https://en.m.wikipedia.org/wiki/Interesting_number_paradox
       | 
       | I suspect that the resolution boils down to what it means to
       | "describe" a number.
        
         | pavlov wrote:
         | I'm not a mathematician but I wonder why the Lowest
         | Uninteresting Number paradox can't be resolved by simply
         | adjusting the definition of "interesting"? After all that's a
         | purely human concept. If we say a number isn't interesting
         | simply due to its proximity to other interesting numbers, then
         | we can draw the boundary.
         | 
         | It seems a bit like making a list of important cities. Borders
         | are very important to countries, but not every border city is
         | economically or historically important. It's not unreasonable
         | to say that a city must meet other criteria than simply being
         | close to a border.
        
           | mannykannot wrote:
           | Yes... when I first saw this paradox, the author of the
           | article I was reading (maybe Martin Gardner?) pointed out
           | that 'uninteresting' is itself not well-defined.
        
             | tuatoru wrote:
             | The set of uninteresting numbers is probably empty.
        
               | hgsgm wrote:
               | Please define "uninteresting" and "probably".
        
             | kybernetikos wrote:
             | I think the uninteresting number one can be described
             | without worrying about 'interestingness'. It works out as
             | the same as saying something like _the set of numbers that
             | are not the smallest number in any set in some arbitrary
             | collection of sets including this one_.
             | 
             | It's the 'including this one' part that is causing the
             | trouble, not which other sets are included in the arbitrary
             | collection.
        
       | [deleted]
        
       | tomp wrote:
       | This is not intuitive at all.
       | 
       | I have a masters in math. Concrete numbers are irrelevant to me,
       | and their explicit representation is even less relevant. _Any_
       | number can easily be described by _x_.
       | 
       | Btw this is just the standard reason for the existence of the
       | Axiom of Choice. _If_ there exists a procedure to describe
       | _which_ number to choose (i.e. an algorithm /formula), you don't
       | need AC at all.
       | 
       | IMO the only "intuitive" anti-AC argument is, that you can get
       | most of math with Axiom of _Countable_ Choice, while you avoid
       | paradoxes like Banach-Tarski.
        
         | davrosthedalek wrote:
         | Question: While the standard construction of the set for
         | Banach-Tarski requires AOC, is it proven that without AOC,
         | Banach-Tarksi isn't true?
        
           | tomp wrote:
           | Not sure,
           | 
           | but if you replace AC with a weaker axiom, Axiom of
           | _Dependent_ Choice, you can construct a model in which all
           | subsets of reals are measurable (called the _Solovay model_
           | ), which precludes the Banach-Tarski paradox (which requires
           | non-measurable sets).
           | 
           | Could BT be constructed without AC but with other way of
           | ensuring there exist non-measurable sets? No idea.
        
           | Kranar wrote:
           | Depends what you mean by without AOC. If you just mean
           | whether Banach-Tarski can be proven in ZF the answer is no.
           | But you also don't need the full power of AOC to prove
           | Banach-Tarski.
        
           | throwaway81523 wrote:
           | By "without AC" I think you mean "with the negation of AC".
           | It turns out that BT+ZF is equivalent to ZF + a weaker
           | version of AC called the Ultrafilter Lemma, iirc. There is
           | more info in the Wikipedia article about BT. Also, the Hahn-
           | Banach theorem (plus ZF) proves BT.
        
         | pdpi wrote:
         | > you can get most of math with Axiom of Countable Choice
         | 
         | Wait, I think you just made me realise that my intuition for
         | AoC is wrong and actually matches AoCC instead, and that I'd
         | never stopped to consider that the collection of sets you're
         | choosing from could be uncountable. If that's correct, that
         | explains so much.
        
         | NotYourLawyer wrote:
         | How do you feel about this example?
         | 
         | https://www.solipsys.co.uk/new/APointAgainstTheAxiomOfChoice...
        
         | lisper wrote:
         | > If there exists a procedure to describe which number to
         | choose (i.e. an algorithm/formula), you don't need AC at all.
         | 
         | The problem here is not that there does not exist a procedure
         | to describe which number to choose, the problem is that there
         | does not exist any way to describe which number you _chose_.
         | Those are not the same thing.
        
           | civilized wrote:
           | Can you give an example in which these two things are
           | different?
        
             | lisper wrote:
             | I'm not sure I understand what you are asking. One is a
             | description of a _procedure_ and the other is a description
             | of a _number_ so obviously these are two different things.
        
         | mckeed wrote:
         | When you say "Any number can easily be described by x" aren't
         | you already using Choice? And doesn't "let x be one of those
         | numbers from the set of numbers that cannot be described with
         | finite symbols" seem a bit like a paradox?
        
       | [deleted]
        
       | CamperBob2 wrote:
       | An object exists prior to being named. If the act of naming (or
       | "counting") the object is all it takes to move it to a different
       | set, then the paradox vanishes, no?
       | 
       | What does it even mean to refer to a "set" of named/unnamed
       | objects or counted/uncounted numbers?
        
       | bannedbybros wrote:
       | [dead]
        
       | tshaddox wrote:
       | > Here's how: consider the set of numbers that cannot be
       | described using any finite collection of symbols. Such numbers
       | must exist because there are only a countably infinite number of
       | numbers that can be described using a finite collection of
       | symbols, but there are an uncountably infinite number of real
       | numbers.
       | 
       | But wait. It's true that there are only countably many "symbolic
       | descriptions," and uncountably many real numbers, but it doesn't
       | follow that there are _specific_ real numbers which cannot be
       | described using a finite collection of symbols, does it?
       | 
       | You can, of course, have a computer algebra system that includes
       | a symbol for a non-computable number like a Chaitin constant
       | (with a well-defined encoding). Wouldn't it be the case that you
       | could define symbols for _any_ countably infinite set of non-
       | computable numbers?
       | 
       | It doesn't seem to me that there is a _fixed_ set of specific
       | non-computable numbers that cannot be referred to by a symbol.
       | Instead, it's just that any particular countably infinite set of
       | symbols can only refer to a countably infinite set of non-
       | computable numbers.
        
         | hgsgm wrote:
         | That's right, we are talking about non computable numbers, so
         | they can't actually be "specific".
         | 
         | Say you have a no computable number A (Chaitin's constant), and
         | I have an uncomputable number B Chaitin's constant but the the
         | trillionth digit is equall to the most common digit in the
         | infinite decimal expansion of Chaitin's constant). How can you
         | tell if they are the same or different? In general, you can't.
         | So what does "specific" even mean?
        
           | tshaddox wrote:
           | I'm not sure if that's relevant. You're pointing out that
           | ordering of the reals is not computable, which is true, but
           | ordering _of the computable numbers_ isn 't even computable.
           | This isn't relevant to whether a symbol in a computer algebra
           | system can refer to a particular number (whether computable
           | or not).
        
         | rcme wrote:
         | > But wait. It's true that there are only countably many
         | "symbolic descriptions," and uncountably many real numbers, but
         | it doesn't follow that there are specific real numbers which
         | cannot be described using a finite collection of symbols, does
         | it?
         | 
         | It does follow because the set of finite collections of symbols
         | is countable.
        
           | tshaddox wrote:
           | I'm not sure what you mean, because you just repeated the
           | first part of my sentence. The fact that one set is countable
           | and the other is uncountable doesn't mean there are
           | _specific, fixed_ numbers in the uncountable set which cannot
           | be paired with any element in the first set.
           | 
           | Consider the usual example of the natural numbers (countable)
           | and the reals (uncountable). It doesn't follow that there is
           | some _specific, fixed_ real number which cannot be part of a
           | bijection of the naturals and the reals. Name any real number
           | and I can give you a bijection of the naturals and the reals
           | which contains your real number.
        
       | jerf wrote:
       | This is close to why I'm not in love with the Axiom of Choice. I
       | don't _hate_ it, and I don 't "mind" it, but I do think
       | mathematicians are perhaps more comfortable with it than they
       | should be. The reason is, I think proofs of the form:
       | 1. A 132 bit (in the information theory sense) definition of
       | something.         2. A 98 bit definition of something else.
       | 3. A 4 bit procedure that puts them together.         4. A 2883
       | bit object of interest.         5. An uncountably infinite number
       | of bits added to the objects in             3 and 4, with no
       | mechanism for determining any of them even            in theory.
       | 6. Therefore, a 76 bit conclusion of interest.
       | 
       | is just intrinsically more _icky_ to me than mathematicians in
       | general think. You know, mathematically icky. It 's a technical
       | term. There's a step here that sticks out like a sore thumb.
       | 
       | Sure, there's a whole lot of fuzziness around measuring the size
       | of a statement in bits, but no matter how you slice it, you're
       | not going to get away from a certain amount of distinction
       | between the statements that are finite, and probably even fairly
       | small in absolute terms, versus the invocation of infinite
       | numbers of bits of information.
       | 
       | (Note that having a decision procedure to determine something,
       | e.g., "here's the set of the primes", is a _finite_ number of
       | bits, and in fact, not even a very large number of them. The set
       | of the primes is completely computable. It may not be
       | "efficiently" computable but it completely computable. It's not
       | just "infinite sets" in general that cause the issue I complain
       | about in general. The axiom of choice specifically invokes
       | infinite numbers of _bits_ , if not uncountably infinite numbers
       | of bits, out of the ether.)
       | 
       | Now, does that make it "invalid"? No. As the article says, it's
       | an axiom, it is what it is. And I don't go to the extreme of
       | tossing it out and completely going to the belief that
       | constructivism is the only valid mathematics. I just am nowhere
       | near _as_ comfortable with it as mathematicians in general are.
        
         | hgsgm wrote:
         | AoC is a hacky shortcut. It gives right answers when it is
         | supposed to, and wrong answers when they don't care.
         | 
         | (Most, non-constructivist) Mathematicians are comfortable with
         | it, because they don't _care_ that it contradicts physical
         | reality, as long as it doesn 't contradict _itself_. They aren
         | 't physicists.
         | 
         | On the flip side, physics don't care about being mathematically
         | correct, as long their errors approximate reality.
         | 
         | "comfort" isn't really useful here. You say you don't toss it
         | out, so what does your discomfort mean? Is there some result
         | you refuse to believe? Or do you simply prefer to study non-AoC
         | math? I like algebra. That doesn't mean I'm not "comfortable"
         | with geometry existing.
        
         | karpierz wrote:
         | Where is infinite information invoked?
        
           | jerf wrote:
           | The Axiom of Choice is that I can have an infinite number of
           | sets and I can "choose" a value from each of them. A choice
           | from a set is basically what a bit is (give or take the
           | question of probabilistic weighting of the values in the set,
           | which is important in the finite case but can't save you in
           | this case so it's a distraction here). Invoking the Axiom of
           | Choice, which is only interesting in the infinite case, is
           | injecting an infinite number of bits into the proof that uses
           | it, as it involves an infinite number of choices.
           | 
           | At best the invocation has an acceptance procedure, but it
           | does not come with a procedure for making the choice.
           | Probably in most cases it doesn't even have an acceptance
           | procedure, which is to say, if we were presented with the
           | choices (or a finite subset of them, since we can't handle
           | the infinite set) we couldn't even tell if they are
           | "correct". So it is not the case that it's just like the
           | primes case, which I mentioned is distinctly finite in bits
           | even if the set is infinite in cardinality. The Axiom of
           | Choice is an infinite (even uncountably infinite or more)
           | number of selections with no way given of representing that
           | in any finite manner. In many cases the Axiom of Choice is
           | invoked when we can't even define the sets we are choosing
           | from, if not most or all of them.
           | 
           | So, from my computer science, information theory point of
           | view, proofs involving the Axiom of Choice are proceeding
           | along nicely, finite statement here, finite statement there,
           | finite logic done on these finite statements, everything's
           | all nice and finite and comprehensible and then _BANG_ an
           | infinite number of unknownable bits are dumped on me, and
           | then we proceed along like nothing weird has happened and do
           | some more finite logic on finite values based on this
           | infinite pile of unknownable bits. It is what it is, but what
           | it is is strange. I mean, it 's not news to mathematicians
           | that it can be a bit strange, but I think this "infinite
           | number of bits" is one of the _better_ ways to see how
           | strange it is.
           | 
           | Another way to look at it is, the AoC is generally invoked
           | without even a theoretical decision procedure to make the
           | choices. It just has a demonstration that such a choice
           | exists. So it amounts to "and then god chooses a value in
           | these sets for us in accordance to our criteria". Lowercase
           | on purpose. A more conventional term in computer science is
           | "oracle". Every invocation of the AoC is basically adding
           | this mathematical entity of such incredible power that we
           | can't determine anything about how it does its thing, or
           | whether or not it is correct. Again. Legal. But something I
           | think is taken more casually than I would like, and I think
           | doing mathematics without scattering infinitely knowledgeable
           | (if only in very constrained ways) and powerful oracles
           | around everywhere with the same casualness as one might do a
           | disjunction or implication in all our proofs is
           | underrespected as a discipline within mathematics in general.
        
             | karpierz wrote:
             | Let's see if I can clear this up:
             | 
             | An equivalent way to phrase the Axiom of Choice is: there
             | exists a function that takes in any non-empty set, and
             | returns an element of that set.
             | 
             | IE, this exists: choose a :: NonEmptySet a -> a
             | 
             | That's it. All of this stuff about "take a choice from a
             | countable number (IE, list) of sets" is fluff; once you
             | have choose, you can make a function that takes in an
             | Iterator (NonEmptySet a) and produces (NonEmptySet a), such
             | that the output will contain every element.
             | 
             | oneFromEachSet :: Iterator (NonEmptySet a) -> NonEmptySet a
             | 
             | oneFromEachSet iter = NonEmptySet (choose (first iter)) <>
             | (oneFromEachSet (rest iter))
             | 
             | Where '<>' is set union.
             | 
             | Does that fix your issue with an "infinite" number of
             | choices? Or do you take issue with assuming 'choose'
             | exists, now that it's a single function?
        
               | jsmith45 wrote:
               | From a programmatic viewpoint that would be describing
               | the axiom of countable choice, since such recursion can
               | only occur countably infinite number of times.
               | 
               | Even if we ignore that by allowing for syntax where we
               | fanout uncountably many times with each recursion, the
               | construction as shown requires the collection of sets to
               | be orderable, to be able to do have a "first", unless we
               | are implementing first in terms of choose itself.
               | (Alternatively one could also argue in terms of Axiom of
               | Choice implying the well-ordering theorem, allowing up to
               | pick a minimum via some such ordering (although you may
               | need to use the choice function to pick one such
               | ordering), but this won't fully help here anyway).
               | 
               | But even if you do that (implement iterator's first in
               | terms of choose) it is not immediately obvious that you
               | really have covered everything some full statements of
               | the axiom of choice does. The axiom of choice is
               | generally phased in terms of collections or families of
               | sets, not sets of sets, but obviously the choose function
               | takes in a set, not a more general family or collection.
               | 
               | It is easy enough to dismiss away certainly non-set
               | collections of sets, like multisets of sets, since
               | "choose" being a function will return the same value
               | every time from a repeated input, and in the above
               | formulation we are returning a set of results, so such
               | duplicates won't matter. We could even handwave away a
               | version where we return a multiset without too much
               | trouble.
               | 
               | But the more expansive versions of the axiom of choice
               | allow us to do something like "choose one element from
               | each set in the class of non-empty sets." This is a real
               | problem, as the "class of non-empty sets" is pretty well
               | known to not be a set, and thus trying to implement
               | iterator first in terms of it would not even be possible
               | for that scenario.
        
               | karpierz wrote:
               | "First" in this case was just getting the first element
               | of the iterator, it wasn't related to choosing a value
               | from the set.
               | 
               | The Axiom of Choice is equivalent to the following:
               | 
               | 1) "For any set A, there exists a function from non-empty
               | elements of the power set of A to elements of A."
               | 
               | Which is equivalent to:
               | 
               | 2) "There exists a function that takes a non-empty set as
               | input, and returns an element of that set"
               | 
               | 2 -> 1, since we can take the function 2 defines, and use
               | it to define the function from 1.
               | 
               | 1 -> 2 since we can define 2 as such:
               | 
               | For a set A, take the power set of A. Then take the
               | function that 1 claims exists. Use that function to get
               | an element of A, since A is an element of its power set.
               | 
               | But back to your core point:
               | 
               | But I agree that Iterator is a bad fit for the axiom of
               | choice, since we have an (uncountable) number of sets to
               | pick from.
               | 
               | Here's a way to phrase it programmatically that allows
               | for uncountable sets-of-sets:
               | 
               | The axiom of choice claims that given a set of non-empty
               | sets, you can construct a new set which contains an
               | element from each of the sets.
               | 
               | pickOneFromEach :: Set (NonEmptySet a) -> NonEmptySet a
               | 
               | If you have a function:
               | 
               | choose :: NonEmptySet a -> a
               | 
               | And a function:
               | 
               | map :: (a -> b) -> Set a -> Set b
               | 
               | then you can define pickOneFromEach as follows:
               | 
               | pickOneFromEach setOfSets = map choose setOfSets
        
               | [deleted]
        
       | otikik wrote:
       | You can say:
       | 
       | "The first number that can't be described using a finite number
       | of symbols"
       | 
       | But then you have just used a finite number of symbols to
       | describe it, haven't you?
        
         | karpierz wrote:
         | This assumes your numbers are well ordered (IE, any set of
         | numbers has a "first"), which is called Zorn's Lemma. It's
         | equivalent to the axiom of choice.
        
         | WastingMyTime89 wrote:
         | You can't say that unambiguously, no. You haven't defined a
         | well order which would allow you to talk about a first element
         | (well, smallest).
        
         | psychoslave wrote:
         | Saying something doesn't imply saying something about a
         | denotable entity.
        
       | IngoBlechschmid wrote:
       | You do not need to worry about the axiom of choice because:
       | 
       | (1) Provably so in a very weak (and hence trustworthy)
       | metatheory, assuming the axiom of choice does not result in any
       | new inconsistencies. More precisely: The systems ZFC (Zermelo-
       | Fraenkel set theory with the axiom of choice) and ZF (ZFC but
       | without choice) are equiconsistent; if ZF is consistent, then so
       | is ZFC.
       | 
       | (2) There is a mechanical procedure for transforming ZFC-proofs
       | of number-theoretic statements into ZF-proofs. That is, appeals
       | to the axiom of choice can always be mechanically eliminated from
       | proofs of number-theoretic statements, even if those proofs rely
       | on results in other areas which crucially depend on the axiom of
       | choice.
       | 
       | You should worry about the axiom of choice because:
       | 
       | (3) Assuming the axiom of choice imposes an undue restriction on
       | the scope of your results: Any theorem proven without the axiom
       | of choice and without the law of excluded middle (which is
       | implied by the axiom of choice) automatically also applies to
       | continuous families. For instance, the theorem "every real
       | symmetric matrix has an eigenvalue" has a proof avoiding AC and
       | LEM. Hence it also holds that for every continuous family of
       | symmetric matrices, locally on the parameter space, there is a
       | continuous eigenvalue-picking function.
       | 
       | PS: One could also wonder whether one should worry about the law
       | of excluded middle. One should not because: (1') The systems ZF
       | and IZF (ZF but without LEM) are equiconsistent. (2') There is a
       | mechanical procedure for transforming ZF-proofs of number-
       | theoretic statements of the special kind "for every numer x,
       | there exists a number y such that some equation holds" into IZF-
       | proofs. On the other hand, one should worry because of (3).
       | 
       | PPS: The axiom which one should truly worry about is the
       | inconspicuous powerset axiom. In contrast with ZFC, ZF and IZF,
       | which are all equiconsistent, removing the powerset axiom results
       | in a drastically weaker system. The proof-theoretic strength of
       | Kripke-Platek set theory (ZF without powerset) is a reasonably
       | large ordinal, whereas we are lacking the technology to even
       | fathom the ordinal calibrating the strength of IZF/ZF/ZFC.
        
         | hackandthink wrote:
         | I am confused:
         | 
         | "From a proof-theoretic perspective, Heyting's calculus is a
         | restriction of classical logic in which the law of excluded
         | middle and double negation elimination have been removed" (1)
         | 
         | "Intuitionistic logic can be understood as a weakening of
         | classical logic, meaning that it is more conservative in what
         | it allows a reasoner to infer" (1)
         | 
         | How is this compatible with:
         | 
         | "The systems ZF and IZF (ZF but without LEM) are
         | equiconsistent." ?
         | 
         | (1) https://en.wikipedia.org/wiki/Intuitionistic_logic
         | 
         | Seems to be quite subtle (proof theoretic strength and
         | consistency strength)
         | 
         | (2)
         | https://mathoverflow.net/questions/126002/interpretability-a...
        
           | librexpr wrote:
           | You might want to check out the Godel-Gentzen negative
           | translation[0], which is an interpretation of classical logic
           | in intuitionistic logic, which can be (somewhat inaccurately)
           | summarized as "if P is provable in classical logic, then not
           | not P is provable in intuitionistic logic".
           | 
           | [0] https://en.wikipedia.org/wiki/G%C3%B6del%E2%80%93Gentzen_
           | neg...
        
           | drdeca wrote:
           | I think "equiconsistent" just means that (one can prove that)
           | if one is consistent, then the other is also consistent.
           | 
           | Doesn't mean the collections of things they can prove can't
           | differ.
           | 
           | For a weak example, aiui, a formal system is equiconsistent
           | with any conservative extension of it, but the conservative
           | extension can (in the cases one would usually speak of) prove
           | some statements that aren't in the language of the original
           | system.
        
             | hackandthink wrote:
             | Thanks, I'm not confused anymore.
        
       | zaroth wrote:
       | You can't say you can't chose an item from a set because you
       | can't describe the item.
       | 
       | If it is sufficiently impossible to describe the number, how did
       | it get _in_ the set.
       | 
       | Irrational numbers are not impossible to identify symbolically,
       | so if you stuff them in a bag and ask me to pick one, I'll just
       | take sqrt(2) thanks.
        
         | vintermann wrote:
         | > If it is sufficiently impossible to describe the number, how
         | did it get in the set.
         | 
         | It got added along with infinitely many others!
         | 
         | That's the thing, it's no problem dealing with "hairy" reals as
         | long as you deal with infinitely many of them at a time. You
         | can say, "all the numbers between zero and one", and the fact
         | that it will include numbers whose decimal expansion can't be
         | described by any rule shouldn't cause any trouble.
         | 
         | But if I ask you, "show me _one_ of these numbers whose decimal
         | expansion can 't be described by any rule", then by definition,
         | you can't.
        
         | hgsgm wrote:
         | How did God create universe?
         | 
         | Does your (non) answer imply that Universe doesn't exist?
        
         | quchen wrote:
         | > You can't say you can't chose an item from a set because you
         | can't describe the item.
         | 
         | Or rather: the whole point of the axiom is giving you such a
         | description.
        
       | heinrichhartman wrote:
       | The axiom of choices is about picking elements of infinitely many
       | sets at once.
       | 
       | In the example we are talking about a single set. The deduction
       | "the set is non-empty => we can pick an element" is much more
       | basic than AC.
        
         | nhatcher wrote:
         | Yeah, that is not the axiom of choice and even if it were their
         | conclusion is not correct. You can't characterize a set as non
         | being described by finite symbols.
         | 
         | On the other hand, it's always refreshing to see someone
         | pushing us to think in this constructs.
         | 
         | Logic is hard!
        
         | lheck wrote:
         | True. It's always good to see someone thinking about math
         | concepts in critical ways to understand them. But this post
         | doesn't really have a place here on hacker news as it's just
         | born of faulty premises and of misunderstanding what the Axiom
         | of Choice states.
         | 
         | The actual HARD PART about the axiom of choice is that the
         | thing f that picks from the collection of sets to its elements
         | that has to exist must be a function. The axiom of choice does
         | not have any reference to f being "described using any finite
         | collection of symbols", or "computable". f must not be
         | computable or graspable or describable, these are not
         | constraints put onto f by the Axiom of Choice. But it must be a
         | function!
         | 
         | Why is "being a function" a constraint for f at all? Not
         | everything that goes from set A to set B is a function.
         | Functions have to be able to be described in terms of sets, and
         | not every collection of items is a set. The notion that every
         | collection is a set is called naive set theory, and it does not
         | work (see Russel's paradox). This is the hard part about the
         | axiom of choice.
         | 
         | So if someone invokes some argument over complex concepts about
         | computability, or being stateable, that's not even true of
         | stuff outside of the Axiom of Choice and definitely not why
         | specifically the AoC is hard.
        
           | quchen wrote:
           | Thanks for pointing out that the _f must be a function_ part
           | is particularly difficult. Is this only a problem for
           | infinite sets (of sets to pick from)? Is it trivial to
           | construct a function that picks a single element from an
           | arbitrary set that's not ordered numbers (say, the real
           | endomorphisms)?
        
       | bell-cot wrote:
       | Reaction: "Intuition" works about as well with the Axiom of
       | Choice as it does with brain surgery, nuclear physics, and rocket
       | science.
        
       | [deleted]
        
       | bmitc wrote:
       | The axiom of choice is not a definition or theorem or problem. It
       | is an axiom. You either take it or leave it.
        
       | auggierose wrote:
       | > Here's how: consider the set of numbers that cannot be
       | described using any finite collection of symbols. Such numbers
       | must exist because there are only a countably infinite number of
       | numbers that can be described using a finite collection of
       | symbols, but there are an uncountably infinite number of real
       | numbers.
       | 
       | But there are uncountably many finite collections of symbols.
       | Sorry, you will have to take the axiom of choice out of my dead
       | cold hands. Banach-Tarski just tells us if we use un-measurable
       | sets, they don't have much to do with our understanding of
       | physical space. But that's fine, because sets are just
       | collections, and physical space is undoubtedly more than that.
        
         | lisper wrote:
         | > there are uncountably many finite collections of symbols
         | 
         | Only if there are uncountably many symbols. But this is a fair
         | point. I'll need to define what I mean by "symbol" more
         | clearly.
        
         | karpierz wrote:
         | There are countably many finite collections of symbols.
         | 
         | There are uncountably many countable collections of symbols.
         | 
         | Edit:
         | 
         | The above is true if your symbols are countable. If they are
         | uncountable, then obviously there are uncountable finite sets
         | of your symbols (As the singleton sets are a subset, and those
         | are uncountable).
        
         | sebzim4500 wrote:
         | >But there are uncountably many finite collections of symbols
         | 
         | How do you mean? Because there are uncountably many symbols?
         | I'm not really sure what a symbol means in this context.
        
           | auggierose wrote:
           | I just picked on one of the points in the post that make no
           | sense without further explanation. Just take each real number
           | as a symbol.
        
             | davrosthedalek wrote:
             | It's normally assumed that the number of symbols is finite.
        
           | ceceron wrote:
           | The number of symbols in the language may countable, but the
           | number of sets of symbols will be indeed uncountable, i.e.
           | AFAIR the number of sets of natural numbers is uncountable --
           | you can even construct real numbers as sets on rational
           | (still countable) numbers.
        
             | [deleted]
        
             | drdeca wrote:
             | The set of all sets of natural numbers is uncountable, but
             | the set of all _finite_ sets of natural numbers, is
             | countable.
        
               | ceceron wrote:
               | True, that's why I haven't used 'finite' in my comment.
               | 
               | Apparently I've missed that the parent referred to the
               | finite collections and in fact was wrong making my
               | previous comment misplaced. Thanks for pointing it out :)
        
         | shellothere wrote:
         | There are countably many finite strings of a fixed, finite
         | collection of symbols. That's the point the author is making --
         | it becomes more intuitively obvious that there might be trouble
         | "choosing" when you design the set you'd like to choose from to
         | consist solely of the "nameless" reals.
        
         | drdeca wrote:
         | There are uncountably many _(countably) infinite_ collections
         | of symbols from within a countably infinite set of possible
         | symbols.
         | 
         | The set of finite subsets of a countably infinite set, is
         | countably infinite.
         | 
         | There is a computable bijection between the set of natural
         | numbers and the set of finite sequences of natural numbers.
        
         | BeetleB wrote:
         | > But there are uncountably many finite collections of symbols.
         | 
         | As others have pointed out, that depends on how big the set of
         | possible symptoms are.
         | 
         | If the set is finite, then there are a countable number of
         | finite collections (the countable union of countable sets is
         | still countable).
         | 
         | If the set is countably infinite, then there are still
         | countably many _finite_ collections.[1]
         | 
         | If the set is uncountably infinite, then you're right. However,
         | the author is probably talking about all symbols known to
         | humanity, which clearly is not only not uncountable, but it is
         | finite.
         | 
         | [1] https://math.stackexchange.com/questions/200389/show-that-
         | th...
        
       | jonathanstrange wrote:
       | I don't think we could talk about numbers that cannot be
       | described by finitely many symbols (let alone a countably
       | infinite number of symbols).
        
       ___________________________________________________________________
       (page generated 2023-01-23 23:02 UTC)